Packrat Cardio Questions

October 28, 2017 | Author: crystalshe | Category: Cardiology, Heart, Hypertension, Heart Failure, Heart Valve
Share Embed Donate


Short Description

physician assistant packrat questions, cardio section...

Description

CARDIO PACKRAT Study online at quizlet.com/_23pihf 1.

History & Physical/Cardiology Which of the following conditions would cause a positive Kussmaul's sign on physical examination? Answers A. Left ventricular failure B. Pulmonary edema C. Coarctation of the aorta D. Constrictive pericarditis

Explanations (u) A. Left ventricular failure results in the back-up of blood into the left atrium and then the pulmonary system so it would not be associated with Kussmaul's sign. (u) B. Pulmonary edema primarily results in increased pulmonary pressures rather than having effects on the venous inflow into the heart. (u) C. Coarctation of the aorta primarily affects outflow from the heart due to the stenosis resulting in delayed and decreased femoral pulses; it has no effect on causing Kussmaul's sign. (c) D. Kussmaul's sign is an increase rather than the normal decrease in the CVP during inspiration. It is most often caused by severe right-sided heart failure; it is a frequent finding in patients with constrictive pericarditis or right ventricular infarction.

2.

History & Physical/Cardiology Anginal chest pain is most commonly described as which of the following? Answers A. Pain changing with position or respiration B. A sensation of discomfort C. Tearing pain radiating to the back D. Pain lasting for several hours

Explanations (u) A. Pain changing with position or respiration is suggestive of pericarditis. (c) B. Myocardial ischemia is often experienced as a sensation of discomfort lasting 5-15 minutes, described as dull, aching or pressure. (u) C. Tearing pain with radiation to the back represents aortic dissection. (u) D. Chest pain lasting for several hours is more suggestive for myocardial infarction.

3.

History & Physical/Cardiology Eliciting a history from a patient presenting with dyspnea due to early heart failure the severity of the dyspnea should be quantified by Answers A. amount of activity that precipitates it. B. how many pillows they sleep on at night. C. how long it takes the dyspnea to resolve. D. any associated comorbidities.

Explanations (c) A. The amount of activity that precipitates dyspnea should be quantified in the history. (u) B. Orthopnea or paroxysmal nocturnal dyspnea can be quantified by how many pillows a patient needs to sleep on to be comfortable. (u) C. How long dyspnea takes to resolve or associated comorbidities has no bearing on quantifying the severity of dyspnea. (u) D. See answer C above.

4.

History & Physical/Cardiology A 25 year-old female presents with a three-day history of chest pain aggravated by coughing and relieved by sitting. She is febrile and a CBC with differential reveals leukocytosis. Which of the following physical exam signs is characteristic of her problem? Answers A. Pulsus paradoxus B. Localized crackles C. Pericardial friction rub D. Wheezing

Explanations (u) A. Pulsus paradoxus is a classic finding for cardiac tamponade. (u) B. Localized crackles are associated with pneumonia and consolidation, not pericarditis. (c) C. Pericardial friction rub is characteristic of an inflammatory pericarditis. (u) D. Wheezing is characteristic for pulmonary disorders, such as asthma.

5.

History & Physical/Cardiology A 65 year-old white female presents with dilated tortuous veins on the medial aspect of her lower extremities. Which of the following would be the most common initial complaint? Answers A. Pain in the calf with ambulation B. Dull aching heaviness brought on by periods of standing C. Brownish pigmentation above the ankle D. Edema in the lower extremities

Explanations (u) A. Patients with deep venous thrombosis (DVT) may present with complaints of pain in the calf with ambulation. Secondary varicosities may result from DVT's. (c) B. Dull aching heaviness or a feeling of fatigue brought on by periods of standing is the most common complaint of patients presenting initially with varicosities. (u) C. Stasis Dermatitis and edema are most suggestive of chronic venous insufficiency. (u) D. See C for explanation.

6.

History & Physical/Cardiology A 22 year-old male received a stab wound in the chest an hour ago. The diagnosis of pericardial tamponade is strongly supported by the presence of Answers A. pulmonary edema. B. wide pulse pressure. C. distended neck veins. D. an early diastolic murmur.

Explanations (u) A. Pulmonary edema may result with low output states as seen with myocardial contusions, but it is not strongly suggestive of tamponade. (u) B. Wide pulse pressure is seen in conditions of high stroke volume such as aortic insufficiency or hyperthyroidism. Narrow pulse pressure is seen with cardiac tamponade. (c) C. Cardiac compression will manifest with distended neck veins and cold clammy skin. (u) D. The onset of diastolic murmur is suggestive of valvular disease, not tamponade.

7.

Diagnostic Studies/Cardiology Cardiac nuclear scanning is done to detect Answers A. electrical conduction abnormalities. B. valvular abnormalities. C. ventricular wall dysfunction. D. coronary artery patency/occlusion.

Explanations (u) A. An EKG is used to determine electrical conduction abnormalities. (u) B. An echocardiogram is a non-invasive test used to determine valvular abnormalities and wall motion. (c) C. Visualization of the cardiac wall can be done with cardiac nuclear scanning. This is done to determine hypokinetic areas from akinetic areas. (u) D. Patency or occlusion is assessed with cardiac catheterization (invasive).

8.

Diagnostic Studies/Cardiology A 72 year-old male with a new diagnosis of congestive heart failure and atrial fibrillation, develops episodes of hemodynamic compromise secondary to increased ventricular rate. A decision to perform elective cardioversion is made and the patient is anticoagulated with heparin. Which test should be ordered to assess for atrial or ventricular mural thrombi?

(u) A. Electrical conduction will not assess for mural thrombi. (u) B. A chest x-ray will not visualize the left atria and ventricles to assess for mural thrombi. (c) C. Transesophageal echocardiography allows for determination of mural thrombi that may have resulted from atrial fibrillation. (u) D. C-reactive protein is not going to give you any information regarding thrombi. This test is used to identify the presence of inflammation.

A. Electrocardiogram B. Chest x-ray C. Transesophageal Echocardiogram D. C-reactive protein 9.

Diagnostic Studies/Cardiology A 64 year-old patient with known history of type 1 diabetes mellitus for 50 years has developed pain radiating from the right buttock to the calf. Patient states that the pain is made worse with walking and climbing stairs. Based upon this history which of the following would be the most appropriate test to order? Answers A. Venogram B. Arterial duplex scanning C. X-ray of the right hip and L/S spine D. Venous Doppler ultrasound

Explanations (u) A. See B for explanation. (c) B. Given the patient's long history of type 1 diabetes mellitus the patient most likely has vascular occlusive disease. Evaluation of arterial blood flow is assessed using the duplex scanner. X-ray of the L/S spine and right hip while not harmful may give information regarding bony structures. Venous Doppler ultrasound will not give information of arterial perfusion. (u) C. See B for explanation. (u) D. See B for explanation.

10.

Diagnostic Studies/Cardiology A 36 year-old male complains of occasional episodes of "heart fluttering". The patient describes these episodes as frequent, short-lived and episodic. He denies any associated chest pain. Based on this information, which one of the following tests would be the most appropriate to order? Answers A. Holter monitor B. Cardiac catheterization C. Stress testing D. Cardiac nuclear scanning

Explanations (c) A. Holter monitoring is a non-invasive test done to obtain a continuous monitoring of the electrical activity of the heart. This can help to detect cardiac rhythm disturbances that can correlate with the patient symptoms. Cardiac catheterization is an invasive procedure done to assess coronary artery disease. Stress testing and cardiac nuclear scanning are non-invasive testing maneuvers done to assess coronary artery disease. (u) B. See A for explanation. (u) C. See A for explanation. (u) D. See A for explanation.

11.

Diagnostic Studies/Cardiology A patient with a mitral valve replacement was placed postoperatively on warfarin (Coumadin) for anticoagulation prophylaxis. To monitor this drug for its effectiveness, what test would be used? Answers A. PTT B. PT-INR C. Platelet aggregation D. Bleeding time

Explanations (u) A. PTT is a reflection of the intrinsic clotting system and is used to monitor heparin administration. (c) B. PT-INR is a reflection of the extrinsic and common pathway clotting system. Coumadin interferes with Vitamin K synthesis which is needed in the manufacture of factors II, VII, IX, X which are part of the extrinsic clotting pathway. (u) C. Platelet aggregation tests are utilized to assess platelet dysfunction. (u) D. Bleeding time is used to assess platelet function.

12.

Diagnosis/Cardiology A 64 year-old male, with a long history of COPD, presents with increasing fatigue over the last three months. The patient has stopped playing golf and also complains of decreased appetite, chronic cough and a bloated feeling. Physical examination reveals distant heart sounds, questionable gallop, lungs with decreased breath sounds at lung bases and the abdomen reveals RUQ tenderness with the liver two finger-breadths below the costal margin, the extremities show 2+/4+ pitting edema. Labs reveal the serum creatinine level 1.6 mg/dl, BUN 42 mg/dl, liver function test's mildly elevated and the CBC to be normal. Which of the following is the most likely diagnosis? Answers A. Right ventricular failure B. Pericarditis C. Exacerbation of COPD D. Cirrhosis

Explanations (c) A. Signs of right ventricular failure are fluid retention i.e. edema, hepatic congestion and possibly ascites. (u) B. See A for explanation. (u) C. See A for explanation. (u) D. See A for explanation.

13.

Diagnosis/Cardiology A 56 year-old male with a known history of polycythemia suddenly complains of pain and paresthesia in the left leg. Physical examination reveals the left leg is cool to the touch and the toes are cyanotic. The popliteal pulse is absent by palpation and Doppler. The femoral pulse is absent by palpation but weak with Doppler. The right leg and upper extremities has 2+/4+ pulses throughout. Given these findings what is the most likely diagnosis? Answers A. Venous thrombosis B. Arterial thrombosis C. Thromboangiitis obliterans D. Thrombophlebitis

Explanations (u) A. See B for explanation. (c) B. Arterial thrombosis has occurred and is evidenced by the loss of the popliteal and dorsalis pedis pulse. This is a surgical emergency. Venous occlusion and thrombophlebitis do not result in loss of arterial pulse. (u) C. See B for explanation. (u) D. See B for explanation.

14.

Diagnosis/Cardiology A 48 year-old male with a known history of hypertension is brought to the ED complaining of headache, general malaise, nausea and vomiting. The patient currently takes nifedipine (Procardia)90mg XL every day and atenolol (Tenormin) 50 mg every day. Vital signs reveal temperature 98.6°F, pulse 72/minute, respiratory rate 20/minute, and the blood pressure is 168/120 mmHg. BP reading taken every 15 minutes from the time of admission reveal the systolic to run from 176 to 186 mmHg and the diastolic to run from 135 to 150 mmHg. Physical examination reveals papilledema bilaterally. There are no renal bruits noted. The EKG is normal. Based upon this presentation, what is the most likely diagnosis? Answers A. Meningitis B. Secondary hypertension C. Pseudotumor cerebri D. Malignant hypertension

Explanations (u) A. See D for explanation. (u) B. See D for explanation. (u) C. Pseudotumor cerebri presents with papilledema, but not hypertension and is more common in young females. (c) D. Malignant HTN is characterized by diastolic reading greater than 140 mm Hg with evidence of target organ damage.

15.

Diagnosis/Cardiology A 55 year-old male is seen in follow-up for a complaint of chest pain. Patient states that he has had this chest pain for about one year now. The patient further states that the pain is retrosternal with radiation to the jaw. "It feels as though a tightness, or heaviness is on and around my chest". This pain seems to come on with exertion however, over the past two weeks he has noticed that he has episodes while at rest. If the patient remains non- active the pain usually resolves in 15-20 minutes. Patient has a 60-pack year smoking history and drinks a martini daily at lunch. Patient appears overweight on inspection. Based upon this history what is the most likely diagnosis? Answers A. Acute myocardial infarction B. Prinzmetal variant angina C. Stable angina D. Unstable angina

Explanations (u) A. Pain does not resolve in an acute MI, it gradually gets worse. (u) B. Pain typically occurs at rest is one of the hallmarks of Prinzmetal variant angina. This patient has just started to develop pain at rest. (u) C. Pain in stable angina is relieved with rest and usually resolves within 10 minutes. angina does not have pain at rest. (c) D. Pain in unstable angina is precipitated by less effort than before or occurs at rest.

16.

Diagnosis/Cardiology Stable A 60 year-old male is brought to the ED complaining of severe onset of chest pain and intrascapular pain. The patient states that the pain feels as though "something is ripping and tearing". The patient appears shocky; the skin is cool and clammy. The patient has an impaired sensorium. Physical examination reveals a loud diastolic murmur and variation in blood pressure between the right and left arm. Based upon this presentation what is the most likely diagnosis? Answers A. Aortic dissection B. Acute myocardial infarction C. Cardiac tamponade D. Pulmonary embolism

Explanations (c) A. The scenario presented here is typical of an ascending aortic dissection. In an acute myocardial infarction the pain builds up gradually. Cardiac tamponade may occur with a dissection into the pericardial space; syncope is usually seen with this occurrence. Pulmonary embolism is usually associated with dyspnea along with chest pain. (u) B. See A for explanation. (u) C. See A for explanation. (u) D. See A for explanation.

17.

Diagnosis/Cardiology A 42 year-old male is brought into the ED with a complaint of chest pain. The pain comes on suddenly without exertion and lasts anywhere from 10-20 minutes. The patient has experienced this on three previous occasions. Today the patient complains of light- headedness with the chest pain lasting longer. Vital signs T-99.3°F oral, P-106/minute and regular, R-22/minute, BP 146/86 mm Hg. EKG reveals sinus rhythm with a rate of 100. Intervals are PR = 0.06 seconds, QRS = 0.12 seconds. A delta wave is noted in many leads. Based upon this information what is the most likely diagnosis? Answers A. Sinus tachycardia B. Paroxysmal supraventricular tachycardia C. Wolff-Parkinson-White syndrome D. Ventricular tachycardia

Explanations (u) A. See C for explanation. (u) B. See C for explanation. (c) C. Wolff-Parkinson-White syndrome hallmarks on EKG include a shorten PR interval, widened QRS, and delta waves. Sinus tachycardia has a normal PR interval and no delta waves. PSVT usually has a retrograde P wave or it may be buried in the QRS complex. (u) D. Ventricular tachycardia has a widened QRS as it originates in the ventricles.

18.

Diagnosis/Cardiology A 63 year-old male is admitted to the hospital with an exacerbation of COPD. The electrocardiogram shows an irregularly, irregular rhythm at a rate of 120/minute with at least three varying P wave morphologies. These electrocardiogram findings are most suggestive of Answers A. atrial fibrillation. B. multifocal atrial tachycardia. C. atrioventricular junctional rhythm. D. third degree heart block.

Explanations (u) A. Atrial fibrillation is an irregularly, irregular rhythm with no definable P waves. (c) B. Multifocal atrial tachycardia is seen most commonly in patients with COPD. Electrocardiogram findings include an irregularly, irregular rhythm with a varying PR interval and various P wave morphologies (Three or more foci). (u) C. Atrioventricular junctional rhythm is an escape rhythm, because of depressed sinus node function, with a ventricular rate between 40-60/minute. (u) D. Third degree heart block presents with a wide QRS at a rate less than 50/minute and blocked atrial impulses.

19.

Health Maintenance/Cardiology A 72 year-old female is being discharged from the hospital following an acute anterolateral wall myocardial infarction. While in the hospital the patient has not had any dysrhythmias or hemodynamic compromise. Which of the following medications should be a part of her d/c meds?

Explanations (u) A. Warfarin is not indicated since there is no role for anticoagulation in this patient. (c) B. ACE inhibitors have been shown to decrease left ventricular hypertrophy and remodeling to allow for a greater ejection fraction. (u) C. The patient does not have any dysrhythmias so Lanoxin is not indicated. (u) D. The patient does not have any hemodynamic compromise or indicators of CHF.

A. Warfarin (Coumadin) B. Captopril (Capoten) C. Digoxin (Lanoxin) D. Furosemide (Lasix) 20.

Health Maintenance/Cardiology A 44 year-old male with a known history of rheumatic fever at age 7 and heart murmur is scheduled to undergo a routine dental cleaning. The murmur is identified as an opening snap murmur. Patient has no known drug allergies. What should this patient receive for antibiotic prophylaxis prior to the dental cleaning? Answers A. This patient does not require antibiotic prophylaxis for a routine dental cleaning. B. This should receive Pen VK 250 mg p.o. QID for 10 days after the procedure. C. This patient should receive Amoxicillin 3.0 gms. p.o. 1 hour before the procedure and then 1.5 gm. 6 hours after the procedure. D. This patient should receive Erythromycin 250 mg QID for 1 day before the procedure and then 10 days after the procedure.

Explanations (h) A. See C for explanation. (u) B. See C for explanation. (c) C. These are the current recommendations from the American Heart Association if the patient is not allergic to penicillin. (u) D. See C for explanation.

21.

Health Maintenance/Cardiology A 36 year-old female presents for a refill of her oral contraceptives. She admits to smoking one pack of cigarettes per day. She should be counseled with regard to her risk of Answers A. venous thrombosis. B. varicose veins. C. atherosclerosis. D. peripheral edema.

Explanations (c) A. Women over age 35 who smoke are at increased risk for the development of venous thrombosis. (u) B. Varicose veins are the result of pressure overload on incompetent veins and not due to the use of oral contraceptives. (a) C. The defined risks of atherosclerosis includes smoking, but does not include the use of oral contraceptives. (u) D. There is no relationship between the use of oral contraceptives and the development of peripheral edema.

22.

Health Maintenance/Cardiology A 68 year-old female comes to the office for an annual physical examination. Her past medical history is significant for a 40-pack year cigarette smoking history. She takes no medications and has not been hospitalized for any surgery. Family medical history reveals that her mother is living, age 87, in good health without medical problems. Her father is deceased at age 45 from a motor vehicle crash. She has two siblings that are alive and well. From this information, how many identifiable risk factors for cardiovascular heart disease exist in this patient? Answers A. 0 B. 1 C. 2 D. 3

Explanations (u) A. See C for explanation. (u) B. See C for explanation. (c) C. This patient has 2 identifiable risk factors based upon the information provided. These include her age 68 and her history of cigarette smoking. (u) D. See C for explanation.

23.

Clinical Intervention/Cardiology Following an acute anterolateral myocardial wall infarction two days ago, a patient suddenly develops hemodynamic deterioration without EKG changes occurring. What complication can explain this scenario? Answers A. Free wall rupture B. CVA C. Atrial fibrillation D. Sick sinus syndrome

Explanations (c) A. Free wall rupture is a complication that occurs within 72 hours of infarction. It is seen mainly in Q wave transmural and lateral wall infarctions. (u) B. See A for explanation. (u) C. See A for explanation. Atrial fibrillation would have EKG evidence of irregularly, irregular rate and rhythm. (u) D. Sick sinus syndrome would have EKG evidence of decreased rate and loss of P waves.

24.

Clinical Intervention/Cardiology A 48 year-old male with a history of coronary artery disease and two myocardial infarctions complains of shortness of breath at rest and 2-pillow orthopnea. His oxygen saturation is 85% on room air. The patient denies any prior history of symptoms. The patient denies smoking. Results of a beta-natriuretic peptide (BNP) are elevated. What should be your next course of action for this patient? Answers A. Send him home on 20 mg furosemide (Lasix) p.o. every day and recheck in one week B. Send him home on clarithromycin (Biaxin) 500 mg p.o. BID and recheck in 1 week C. Admit to the hospital for work up of left ventricular dysfunction D. Admit to the hospital for work up of pneumonia

Explanations (h) A. See C for explanation. (h) B. See C for explanation. (c) C. An elevated BNP is seen in a situation where there is increased pressure in the ventricle during diastole. This is representative of the left ventricle being stretched excessively when a patient has CHF. Sending a patient home would be inappropriate in this case. (u) D. See C for explanation.

25.

Clinical Intervention/Cardiology A 48 year-old male presents to the ED with complaints of chest pressure, dyspnea on exertion, and diaphoresis that has been present for the last one hour. Electrocardiogram reveals normal sinus rhythm at 92/minute along with ST segment elevation in leads V3-V5. Initial cardiac enzymes are normal. What is the next most appropriate step in the management of this patient? Answers A. Coronary artery revascularization B. Admission for medical management C. Administer lidocaine D. Administer nitrates

Explanations (c) A. The standard of care for the management of acute ST-segment elevation MI is coronary artery revascularization. This patient is diagnosed with an STsegment elevation MI based upon his history and EKG findings. Cardiac enzymes are normal because of the early presentation of this patient. (u) B. Although this patient will be admitted to the hospital, this patient needs to have acute management of the myocardial infarction without delay. (h) C. Prophylactic lidocaine has been shown to increase morbidity and mortality from acute MI when used in this setting. (u) D. Although pain control is a goal for patients with acute MI, it is not the essential medication that will impact this patient's care to the greatest degree.

26.

Clinical Intervention/Cardiology An unresponsive patient is brought to the ED by ambulance. He is in ventricular tachycardia with a heart rate of 210 beats/min and a blood pressure of 70/40 mmHg. The first step in treatment is to Answers A. administer IV adenosine. B. DC cardiovert. C. administer IV lidocaine. D. apply overdrive pacer.

Explanations (u) A. Adenosine is used to treat PSVT. (c) B. The first step in treatment of unstable ventricular tachycardia with a pulse is to cardiovert using a 100 J countershock. (u) C. See B for explanation. (u) D. Overdrive pacing is indicated in Torsades de Pointes.

27.

Clinical Therapeutics/Cardiology Which of the following antiarrhythmic drugs can be associated with hyper- or hypothyroidism following long-term use? Answers A. Quinidine B. Amiodarone C. Digoxin D. Verapamil

Explanations (u) A. See B for explanation. (c) B. Amiodarone is structurally related to thyroxine and contains iodine, which can induce a hyper- or hypothyroid state. (u) C. See B for explanation. (u) D. See B for explanation.

28.

Clinical Therapeutics/Cardiology Which of the following hypertensive emergency drugs has the potential for developing cyanide toxicity? Answers A. Sodium nitroprusside (Nipride) B. Diazoxide (Hyperstat) C. Labetalol (Normodyne) D. Alpha-methyldopa (Aldomet)

Explanations (c) A. Sodium nitroprusside metabolization results in cyanide ion production. It can be treated with sodium thiosulfite, which combines with the cyanide ion to form thiocyanate, which is nontoxic. (u) B. See A for explanation. (u) C. See A for explanation. (u) D. See A for explanation.

29.

Clinical Therapeutics/Cardiology Contraindications to beta blockade following an acute myocardial infarction include which of the following? Answers A. Third degree A-V block B. Sinus tachycardia C. Hypertension D. Rapid ventricular response to Atrial fibrillation/flutter

Explanations (c) A. Beta blockade is contraindicated in second and third heart block. (u) B. Beta blockade has been proven to be beneficial in sinus tachycardia, hypertension and in atrial fib/flutter with a rapid ventricular response. (u) C. See B for explanation. (u) D. See B for explanation.

30.

Clinical Therapeutics/Cardiology A 74 year-old male is diagnosed with pneumonia. The physician assistant should ensure the patient is not on which of the following before starting therapy with clarithromycin (Biaxin)? Answers A. Lisinopril (Zestril) B. Furosemide (Lasix) C. Simvastatin (Zocor) D. Dipyridamole (Persantine)

Explanations (u) A. See C for explanation. (u) B. See C for explanation. (c) C. Statins are known to interact with the macrolides as they may cause prolonged QT interval, myopathy and rhabdomyolysis. (u) D. See C for explanation.

31.

Clinical Therapeutics/Cardiology According to the recent JNC VII guidelines, a 34 year-old male who has type 1 diabetes mellitus and hypertension should be started on which type of antihypertensive agent? Answers A. Beta-blocker B. Loop diuretic C. ACE inhibitor D. Thiazide diuretic

Explanations (u) A. Beta blockers could potentially be harmful in a patient with diabetes mellitus. Use a cardioselective betablocker to reduce the incidence of hypoglycemia. (u) B. See C for explanation. (c) C. ACE inhibitors are effective in young patients. They are capable of providing protection to the kidney especially in diabetes mellitus. (u) D. See C for explanation.

32.

Clinical Therapeutics/Cardiology Which of the following beta-adrenergic blocking agents has cardioselectivity for primarily blocking beta-1 receptors? Answers A. Propranolol (Inderal) B. Timolol (Blocadren) C. Metoprolol (Lopressor) D. Pindolol (Visken)

Explanations (u) A. Propranolol and timolol are nonselective betaadrenergic antagonists. (u) B. See A for explanation. (c) C. Metoprolol is selective for beta-1 antagonists (u) D. Pindolol is an antagonist with partial agonist activity.

33.

Scientific Concepts/Cardiology Which of the following is the mechanism of action of Class III antiarrhythmic drugs? Answers A. Na+ channel blocker B. K+ channel blocker C. Beta adrenoreceptor blocker D. Ca++ channel blocker

Explanations (u) A. Na+ channel blockers are Class I. (c) B. K+ channel blockers are Class III. (u) C. Beta adrenoreceptor blockers are Class II. (u) D. Ca++ channel blockers are Class VI.

34.

Scientific Concepts/Cardiology In congestive heart failure the mechanism responsible for the production of an S3 gallop is Answers A. contraction of atria in late diastole against a stiffened ventricle. B. rapid ventricular filling during early diastole. C. vibration of a partially closed mitral valve during mid to late diastole. D. secondary to closure of the mitral valve leaflets during systole.

Explanations (u) A. Atrial contraction against a noncompliant ventricle is the mechanism responsible for S4. (c) B. Rapid ventricular filling during early diastole is the mechanism responsible for the S3. (u) C. Vibration of a partially closed mitral valve during mid to late diastole is the mechanism responsible for the Austin-Flint murmur of aortic regurgitation. (u) D. Closure of the mitral valve leaflets during systole is the mechanism responsible for part of the S1 heart sound.

35.

Scientific Concepts/Cardiology What is the most likely mechanism responsible for retinal hemorrhages and neurologic complications in a patient with infective endocarditis? Answers A. Metabolic acidosis B. Systemic arterial embolization of vegetations C. Hypotension and tachycardia D. Activation of the immune system

Explanations (u) A. See B for explanation. (c) B. The vegetations that occur during infective endocarditis can become emboli and can be dispersed throughout the arterial system. (u) C. See B for explanation. (u) D. Glomerulonephritis and arthritis result from activation of the immune system.

36.

Scientific Concepts/Cardiology During an inferior wall myocardial infarction the signs and symptoms of nausea and vomiting, weakness and sinus bradycardia are a result of what mechanism? Answers A. Increased sympathetic tone B. Increased vagal tone C. Activation of the renin-angiotensin system D. Activation of the inflammatory and complement cascade system

Explanations (u) A. See B for explanation. (c) B. Increased vagal tone is common in inferior wall MI; if the SA node is involved, bradycardia may develop. (u) C. See B for explanation. (u) D. See B for explanation.

37.

Scientific Concepts/Cardiology Which of the following is the most common cause of secondary hypertension? A. Renal parenchymal disease B. Primary aldosteronism C. Oral contraceptive use D. Cushing's syndrome

Explanations (c) A. Renal parenchymal disease is the most common cause of secondary hypertension. (u) B. Primary aldosteronism can cause secondary hypertension, but it is not the most common cause. (u) C. Oral contraceptives can cause small increases in blood pressure but considerable increases are much less common. (u) D. Cushing's disease is a less common cause of secondary hypertension.

38.

Clinical Therapeutics/Cardiology Which of the following medication classes is the treatment of choice in a patient with variant or Prinzmetal's angina? A. Calcium channel blockers B. ACE inhibitors C. Beta blockers D. Angiotensin II receptor blockers

Explanations (c) A. Calcium channel blockers are effective prophylactically to treat coronary vasospasm associated with variant or Prinzmetal's angina. (u) B. ACE inhibitors are not a treatment for coronary vasospasm. (h) C. Beta blockers have been noted to exacerbate coronary vasospasm potentially leading to worsening ischemia. (u) D. Angiotensin II receptor blockers are not a treatment for coronary vasospasm.

39.

Clinical Therapeutics/Cardiology A 63 year-old female with history of diabetes mellitus presents for blood pressure follow-up. At her last two visits her blood pressure was 150/92 and 152/96. Today in the office her blood pressure is 146/92. Recent blood work shows a Sodium 140 mEq/L, Potassium 4.2 mEq/L, BUN of 23 mg/dL, and Creatinine of 1.1 mg/dL. Which of the following is the most appropriate initial medication in this patient? A. Terazosin (Hytrin) B. Atenolol (Tenormin) C. Lisinopril (Zestril) D. Hydrochlorothiazide (HCTZ)

Explanations (u) A. Alpha blockers are not the treatment of choice in a diabetic with hypertension. (u) B. Patients with hypertension and diabetes may require a Beta blocker, but it should be added to an ACE inhibitor if the ACE inhibitor is ineffective on its own. (c) C. ACE inhibitors should be part of the initial treatment of hypertension in diabetics because of beneficial effects in diabetic nephropathy and is the most appropriate initial medication. (u) D. Patients with hypertension and diabetes mellitus may require a diuretic, but it should be added to an ACE inhibitor if the ACE inhibitor is ineffective on its own.

40.

Diagnostic Studies/Cardiology What is the EKG manifestation of cardiac end-organ damage due to hypertension? A. Right bundle branch block B. Left ventricular hypertrophy C. Right ventricular hypertrophy D. ST segment elevation in lateral precordial leads

Explanations (u) A. Right bundle branch block is caused by a delay in the conduction system in the right ventricle. It may be caused by right ventricular hypertrophy or conditions with higher pulmonic resistance such as cor pulmonale. Hypertension, however, is likely to cause changes in the left ventricle rather than the right ventricle. (c) B. Long-standing hypertension can lead to left ventricular hypertrophy with characteristic changes noted on EKG. (u)C. See A for explanation. (u) D. ST segment elevation is a sign of acute myocardial infarction not hypertension.

41.

Health Maintenance/Cardiology Annual blood pressure determinations should be obtained beginning at the age of A. 3 years. B. 5 years. C. 12 years. D. 18 years.

Explanations (c) A. Periodic measurements of blood pressure should be part of routine preventive health assessments beginning at the age of 3 years. (u) B. See A for explanation. (u) C. See A for explanation. (u) D. See A for explanation.

42.

History & Physical/Cardiology Which of the following conditions would cause a positive Kussmaul's sign on physical examination? A. Left ventricular failure B. Pulmonary edema C. Coarctation of the aorta D. Constrictive pericarditis

Explanations (u) A. Left ventricular failure results in the back-up of blood into the left atrium and then the pulmonary system so it would not be associated with Kussmaul's sign. (u) B. Pulmonary edema primarily results in increased pulmonary pressures rather than having effects on the venous inflow into the heart. (u) C. Coarctation of the aorta primarily affects outflow from the heart due to the stenosis resulting in delayed and decreased femoral pulses; it has no effect on causing Kussmaul's sign. (c) D. Kussmaul's sign is an increase rather than the normal decrease in the CVP during inspiration. It is most often caused by severe right-sided heart failure; it is a frequent finding in patients with constrictive pericarditis or right ventricular infarction.

43.

History & Physical/Cardiology Which of the following physical findings is suggestive of atrial septal defect? A. Fixed split S2 B. Increased pulse pressure C. Continuous mechanical murmur D. Difference in blood pressure between the left and right arm

Explanations (c) A. An atrial septal defect will cause a shunt of blood from the left to the right atrium. This will result in an equalization in the amount of blood entering both the left and right ventricles which effectively eliminates the normally wide splitting that inspiration typically causes in hearts without an atrial septal defect. (u) B. Pulse pressures reflect the difference in aortic and left ventricular volumes that occur during ventricular systole Increased pulse pressures are seen in aortic regurgitation which is a different entity than atrial septal defect. (u) C. Continuous mechanical murmurs are noted in patients with patent ductus arteriosus. (u) D. Differences in blood pressure between the left and right arms are seen in conditions such as coarctation of the aorta.

44.

Clinical Therapeutics/Cardiology A 29 year-old male presents with complaint of substernal chest pain for 12 hours. The patient states that the pain radiates to his shoulders and is relieved with sitting forward. The patient admits to recent upper respiratory symptoms. On examination vital signs are BP 126/68, HR 86, RR 20, temp 100.3 degrees F. There is no JVD noted. Heart exam reveals regular rate and rhythm with no S3 or S4. There is a friction rub noted. Lungs are clear to auscultation. EKG shows diffuse ST segment elevation. What is the treatment of choice in this patient? A. Pericardiocentesis B. Nitroglycerin C. Percutaneous coronary intervention D. Indomethacin (Indocin)

Explanations (u) A. Pericardiocentesis is the treatment of choice in a patient with a pericardial effusion and cardiac tamponade, there is no evidence of either of these in this patient. (u) B. Nitroglycerin is indicated in the treatment of chest pain related to angina. (u) C. Percutaneous coronory intervention is the treatment of choice in a patient with an acute myocardial infarction. (c) D. Indomethacin, a nonsteroidal antiinflammatory medication, is the treatment of choice in a patient with acute pericarditis.

45.

Diagnosis/Cardiology A 24 year-old male presents for routine physical examination. On physical examination, you find that the patient's upper extremity blood pressure is higher than the blood pressure in the lower extremity. Heart exam reveals a late systolic murmur heard best posteriorly. What is the most likely diagnosis in this patient? A. Hypertrophic obstructive cardiomyopathy B. Patent foramen ovale C. Coarctation of the aorta D. Patent ductus arteriosus

Explanations (u) A. Patients with hypertrophic obstructive cardiomyopathy do not present with hypertension or weak femoral pulses. (u) B. The murmur associated with patent foramen ovale is a systolic ejection murmur heard in the second and third intercostal spaces and patients do not present with hypertension. (c) C. Coarctation of the aorta commonly presents with higher systolic pressures in the upper extremities than the lower extremities and absent or weak femoral pulses. (u) D. Patent ductus arteriosus is rare in adults and patients are noted to have a continuous rough, machinery murmur.

46.

Diagnosis/Cardiology A 63 year-old female presents with a complaint of chest pressure for one hour, noticed upon awakening. She admits to associated nausea, vomiting, and shortness of breath. 12 lead EKG reveals ST segment elevation in leads II, III, and AVF. Which of the following is the most likely diagnosis? A. Aortic dissection B. Inferior wall myocardial infarction C. Acute pericarditis D. Pulmonary embolus

Explanations (u) A. A patient with aortic dissection will complain of tearing, ripping pain. EKG is often normal, but may reveal left ventricular strain pattern. (c) B. Myocardial infarction often presents with chest pressure and associated nausea and vomiting. ST segment elevation in leads II, III, and AVF are classic findings seen in acute inferior wall myocardial infarction. (u) C. Acute pericarditis presents with atypical chest pain and diffuse ST segment elevation. (u) D. Pulmonary embolism often presents with either no EKG changes or sinus tachycardia. Classically described, rarely seen findings include a large S wave in lead I, a Q wave with T wave inversion in lead III, ST segment depression in lead II, T wave inversion in leads V1-V4 and a transient right bundle branch block.

47.

History & Physical/Cardiology A 12 month-old child with tetralogy of Fallot is most likely to have which of the following clinical features? A. Chest pain B. Cyanosis C. Convulsions D. Palpitations

Explanations (u) A. Chest pain is not a feature of tetralogy of Fallot. (c) B. Cyanosis is very common in tetralogy of Fallot. (u) C. Convulsions are occasionally seen as part of severe hypoxic spells in infancy rather than a feature of tetralogy of Fallot. (u) D. Palpitations are uncommon in tetralogy of Fallot.

48.

Diagnosis/Cardiology A 23 year-old male presents with syncope. On physical examination you note a medium-pitched, mid-systolic murmur that decreases with squatting and increases with straining. Which of the following is the most likely diagnosis? A. Hypertrophic cardiomyopathy B. Aortic stenosis C. Mitral regurgitation D. Pulmonic stenosis

Explanations (c) A. Hypertrophic cardiomyopathy is characterized by a medium- pitched, mid-systolic murmur that decreases with squatting and increases with straining. (u) B. Straining decreases the intensity of the murmur associated with aortic stenosis and squatting increases the intensity. (u) C. Mitral regurgitation is characterized by a blowing systolic murmur that radiates to the axilla, it is not often associated with syncope. (u) D. Pulmonic stenosis is a harsh systolic murmur with a widely split S2, and no change with maneuvers.

49.

Health Maintenance/Cardiology A patient with which of the following is at highest risk for coronary artery disease? A. Congenital heart disease B. Polycystic ovary syndrome C. Acute renal failure D. Diabetes mellitus

Explanations (u) A. Congenital heart disease is not an established risk factor for coronary artery disease. (u) B. While patients with polycystic ovary syndrome have hyperinsulimemia, they do not have the same poor prognosis for coronary artery disease as patients with diabetes mellitus. (u) C. Patients with acute renal failure are not at risk for coronary artery disease, although patients with diabetes and chronic renal disease do have this risk. (c) D. Patients with diabetes mellitus are in the same risk category for coronary artery disease as those patients with established atherosclerotic disease.

50.

Clinical Therapeutics/Cardiology Acute rebound hypertensive episodes have been reported to occur with the sudden withdrawal of A. verapamil (Calan). B. lisinopril (Prinivil). C. clonidine (Catapres). D. hydrochlorothiazide (HCTZ)

Explanations (u) A. Verapamil is a calcium channel blocker and there is no associated rebound hypertension after withdrawal. (u) B. Lisinopril is an ACE inhibitor, which is not associated with rebound hypertension. (c) C. Clonidine (Catapres) is a central alpha agonist and abrupt withdrawal may produce a rebound hypertensive crisis. (u) D. Hydrochlorothiazide is a thiazide diuretic, which is not associated with rebound hypertension.

51.

Diagnosis/Cardiology A 38 year-old female with history of coarctation of the aorta repair at the age of two presents with fevers for four weeks. The patient states that she has felt fatigued and achy during this time. Maximum temperature has been 102.1 degrees F. She denies cough, congestion, or other associated symptoms. Physical examination reveals a pale tired appearing female in no acute distress. Heart reveals a new grade III-IV/VI systolic ejection border at the apex, and a II/VI diastolic murmur at the right sternal border. What is the most likely diagnosis? A. Acute myocardial infarction B. Bacterial endocarditis C. Acute pericarditis D. Restrictive cardiomyopathy

Explanations (u) A. Acute MI presents with complaint of chest pain, SOB, not with fever and myalgias. (c) B. Bacterial endocarditis presents as febrile illness lasting several days to weeks, commonly with nonspecific symptoms, echocardiogram often reveals vegetations on affected valves. (u) C. Pericarditis does not present with systolic or diastolic murmur or vegetation, more commonly pericardial friction rub would be noted. (u) D. Restrictive cardiomyopathy will show impaired diastolic filling on echocardiogram and is not associated with fever.

52.

Diagnostic Studies/Cardiology A 23 year-old female with history of palpitations presents for evaluation. She admits to acute onset of rapid heart beating lasting seconds to minutes with associated shortness of breath and chest pain. The patient states she can relieve her symptoms with valsalva. Which of the following is the most appropriate diagnostic study to establish a definitive diagnosis in this patient? A. Cardiac catheterization B. Cardiac MRI C. Chest CT scan D. Electrophysiology study

Explanations (u) A. Cardiac catheterization evaluates coronary arteries but has no role in the diagnosis of supraventricular tachycardia. (u) B. Cardiac MRI cannot diagnose and define pathway of supraventricular tachycardia. (u) C. Chest CT scan will not establish definitive diagnosis of supraventricular tachycardia. (c) D. Electrophysiology study is useful in establishing the diagnosis and pathway of complex arrhythmias such as supraventricular tachycardia.

53.

Clinical Therapeutics/Cardiology Which of the following is the chief adverse effect of thiazide diuretics? A. Hypokalemia B. Hypernatremia C. Hypocalcemia D. Hypermagnesemia

Explanations (c) A. Thiazide diuretics can induce electrolyte changes. Principle among those is hypokalemia. (u) B. Hyponatremia, not hypernatremia may be a complication of thiazide diuretics. (u) C. Thiazide diuretics cause the retention of calcium and would not cause hypocalcemia. (u) D. Thiazide diuretics cause the retention of calcium and do not readily affect magnesium levels.

54.

Clinical Intervention/Cardiology A 25 year-old male with history of syncope presents for evaluation. The patient admits to intermittent episodes of rapid heart beating that resolve spontaneously. 12 Lead EKG shows delta waves and a short PR interval. Which of the following is the treatment of choice in this patient? A. Radiofrequency catheter ablation B. Verapamil (Calan) C. Percutaneous coronary intervention D. Digoxin (Lanoxin)

Explanations (c) A. Radiofrequency catheter ablation is the treatment of choice on patients with accessory pathways, such as Wolff-Parkinson-White Syndrome. (h) B. Calcium channel blockers such as verapamil decrease refractoriness of the accessory pathway or increase that of the AV node leading to faster ventricular rates, therefore calcium channel blockers should be avoided in patients with WPW. (u) C. Percutaneous coronary intervention is indicated in the treatment of coronary artery disease, not preexcitation syndromes. (h) D. Digoxin decreases refractoriness of the accessory pathway and increases that of the AV node leading to faster ventricular rates. It should therefore be avoided in patients with WPW.

55.

History & Physical/Cardiology A patient presents for a follow-up visit for chronic hypertension. Which of the following findings may be noted on the fundoscopic examination of this patient? A. cherry-red fovea B. boxcar segmentation of retinal veins C. papilledema D. arteriovenous nicking

Explanations (u) A. Cherry-red fovea and boxcar segmentation of the retinal veins are findings seen in central retinal artery occlusion. (u) B. See letter A for explanation. (u) C. Papilledema is noted in conditions causing increased intracranial pressure. (c) D. Arteriovenous nicking is common in chronic hypertension.

56.

Diagnostic Studies/Cardiology Which of the following diagnostic tests should be ordered initially to evaluate for suspected deep venous thrombosis of the leg? A. Venogram B. Arteriogram C. Duplex ultrasound D. Impedance plethysmography

Explanations (u) A. Venogram has been replaced by noninvasive tests due to discomfort, cost, technical difficulties, and complications, such as phlebitis. 56 (h) B. Thrombophlebitis is a venous problem, not an arterial one. Any unnecessary invasive procedure is potentially harmful. (c) C. Ultrasound is the technique of choice to detect deep venous thrombosis in the leg. (a) D. Impedance plethysmography is equivalent to ultrasound in detecting thrombi of the femoral and popliteal veins, but it may miss early, nonocclusive thrombi.

57.

Diagnosis/Cardiology A 36 year-old patient with cardiomyopathy secondary to viral myocarditis develops fatigue, increasing dyspnea, and lower extremity edema over the past 3 days. He denies fever. A chest x-ray shows no significant increase in heart size, but reveals prominence of the superior pulmonary vessels. Based on these clinical findings, which of the following is the most likely diagnosis? A. Heart failure B. Subacute bacterial endocarditis C. Pulmonary embolus D. Pneumonia

Explanations (c) A. Given the presence of cardiomyopathy, the patient's heart has decreased functional reserve. The symptoms and chest x-ray findings are typical of congestive heart failure. (u) B. Endocarditis occurs as a result of infection that primarily occurs in the blood stream. Endocarditis would present with signs of infection or seeding rather than signs of heart failure. (u) C. Pulmonary embolus usually presents with an acute onset of chest pain, severe dyspnea, and anxiety. (u) D. Pneumonia is less likely since there is no fever and edema is not usually associated with pneumonia.

58.

Clinical Intervention/Cardiology Which of the following is first-line treatment for symptomatic bradyarrhythmias due to sick sinus syndrome (SSS)? A. Permanent pacemaker B. Radiofrequency ablation C. Antiarrhythmics D. Anticoagulation therapy

Explanations (c) A. Permanent pacemakers are the therapy of choice in patients with symptomatic bradyarrhythmias in sick sinus syndrome. (u) B. Radiofrequency ablation is used for the treatment of accessory pathways in the heart. (u) C. See A for explanation. (u) D. See A for explanation.

59.

History & Physical/Cardiology What type of chest pain is most commonly associated with a dissecting aortic aneurysm? A. Squeezing B. Dull, aching C. Ripping, tearing D. Burning

Explanations (u) A. Squeezing pain is more characteristic of angina or esophageal pain. (u) B. Dull, aching pain is more characteristic of chest wall pain, possibly angina, or anxiety. (c) C. A dissecting aortic aneurysm often presents with a very severe ripping, tearing-like pain. (u) D. Burning pain is more characteristic of esophageal reflux, esophagitis, or tracheobronchitis.

60.

Health Maintenance/Cardiology A 52 year-old obese female with a history of hypertension, tobacco abuse, and hyperlipidemia presents for routine follow-up. Which of her risk factors for coronary atherosclerosis is not modifiable? A. Age B. High LDL C. Hypertension D. Obesity

Explanations (c) A. Age is a non modifiable risk factor, as is family history of premature coronary heart disease (u) B. High LDL is a modifiable risk factor, as is Hypertension, low HDL, obesity, tobacco abuse, physical inactivity (u) C. See B for explanation. (u) D. See B for explanation.

61.

Diagnosis/Cardiology An 8 year-old boy is brought to a health care provider complaining of dyspnea and fatigue. On physical examination, a continuous machinery murmur is heard best in the second left intercostal space and is widely transmitted over the precordium. The most likely diagnosis is A. ventricular septal defect. B. atrial septal defect. C. congenital aortic stenosis. D. patent ductus arteriosus.

Explanations (u) A. Ventricular septal defect causes a holosystolic murmur rather than a continuous machinery-like murmur. (u) B. Atrial septal defect causes a fixed split S2 rather than a continuous systolic heart murmur. (u) C. Congenital aortic stenosis causes a crescendo-decrescendo systolic murmur heard best in the second intercostal space. (c) D. Patent ductus arteriosus is classically described in children as a continuous machinery-type murmur that is widely transmitted across the precordium.

62.

Diagnostic Studies/Cardiology A 63 year-old male with history of hypertension and tobacco abuse presents complaining of dyspnea on exertion for two weeks. The patient admits to one episode of chest discomfort while shoveling snow which was relieved after five minutes of rest. Vital signs are BP 130/70, HR 68, RR 14. Heart exam reveals regular rate and rhythm, normal S1 and S2, no murmur, gallop, or rub. Lungs are clear to auscultation bilaterally. There is no edema noted. Which of the following is the most appropriate initial diagnostic study for this patient? A. Helical CT scan B. Chest x-ray C. Nuclear stress test D. Cardiac catheterization

Explanations (u) A. Helical CT scan aids in the diagnosis of pulmonary embolism, not in the evaluation of angina. (u) B. Chest x-ray is not used as a diagnostic study to evaluate symptoms of angina or coronary heart disease. (c) C. In patients with classic symptoms of angina, nuclear stress testing is the most widely used test for diagnosis of ischemic heart disease. (u) D. Coronary angiography is indicated in patients with classic stable angina who are severely symptomatic despite medical therapy and are being considered for percutaneous intervention (PCI), patients with troublesome symptoms that are difficult to diagnose, angina symptoms in a patient who has survived sudden cardiac death event, patients with ischemia on noninvasive testings.

63.

Clinical Intervention/Cardiology A 52 year-old male with history of hypertension and hyperlipidemia presents with an acute myocardial infarction. Urgent cardiac catheterization is performed and shows a 90% occlusion of the left anterior descending artery. The other arteries have minimal disease. Ejection fraction is 45%. Which of the following is the treatment of choice in this patient? A. Coronary artery bypass grafting (CABG) B. Streptokinase C. Percutaneous coronary intervention (PCI) D. Warfarin (Coumadin)

Explanations (u) A. Percutaneous coronary intervention is a better, less invasive alternative to CABG for single vessel coronary artery disease. (h) B. Streptokinase is not commonly used for treatment of acute myocardial infarction because it is ineffective at opening the occluded artery and reducing mortality. Streptokinase would be harmful because it would increase the risk of bleeding. (c) C. Immediate coronary angiography and primary percutaneous coronary intervention have been shown to be superior to thrombolysis. (u) D. Warfarin is used to prevent thrombosis and not for acute treatment.

64.

Clinical Intervention/Cardiology A patient presents with an acutely painful and cold left leg. Distal pulses are absent. Leg is cyanotic. There are no signs of gangrene or other open lesions. Symptoms occurred one hour ago. Which of the following treatments is most appropriate? A. Vena cava filter B. Embolectomy C. Amputation D. Aspirin

Explanations (u) A. Vena cava filters are used in the management of venous thromboembolic disease when anticoagulation cannot be done. (c) B. Embolectomy within 4 to 6 hours is the treatment of choice. (h) C. Amputation is done only when no viable tissue is present. Cutting off a viable limb is never a good idea. (u) D. Aspirin is used in the prevention and treatment of coronary disease and has no role in the treatment of peripheral arterial embolism.

65.

Clinical Therapeutics/Cardiology Which of the following medications used in the treatment of supraventricular tachycardia is able to cause sinus arrest and asystole for a few seconds while it breaks the paroxysmal supraventricular tachycardia? A. Digoxin (Lanoxin) B. Adenosine (Adenocard) C. Verapamil (Calan) D. Quinidine (Quinaglute)

Explanations (u) A. Digoxin is not used for the acute termination of supraventricular tachycardia. (c) B. Adenosine is an endogenous nucleoside that results in profound (although transient) slowing of the AV conduction and sinus node discharge rate. This agent has a very short half-life of 6 seconds. (u) C. Although verapamil may be used for the termination of acute supraventricular tachycardia, it does not lead to sinus arrest in therapeutic doses. (u) D. Quinidine is rarely used today and is not indicated for the termination of supraventricular tachycardia.

66.

Diagnosis/Cardiology An elderly female presents for evaluation of exertional syncope, dyspnea, and angina. She admits that previous to these symptoms she had insidious progression of fatigue that caused her to curtail her activities. Which of the following is the most likely diagnosis? A. Aortic stenosis B. Aortic regurgitation C. Mitral stenosis D. Mitral valve prolapse

Explanations (c) A. The major symptoms of aortic stenosis are exertional syncope, dyspnea, and angina. Symptoms do not become apparent for a number of years and usually are not present until the valve is narrowed to less than 0.5 cm to 2 cm of valve surface area. (u) B. Patients with aortic regurgitation are likely to complain of an uncomfortable awareness of their heart, especially when lying down. These patients develop sinus tachycardia with exertion and complain of palpitations and head pounding with activity. (u) C. The symptoms related to mitral stenosis are related to increased pulmonary pressure after the left atrium can no longer overcome the outflow obstruction. (u) D. Patients with mitral valve prolapse are typically asymptomatic throughout their lives, although a wide range of symptoms is possible. When symptoms do occur, palpitations from arrhythmias are most common along with lightheadedness. Syncope is not part of this disease process.

67.

History & Physical/Cardiology Which of the following would you expect on physical examination in a patient with mitral valve stenosis? A. Systolic blowing murmur B. Opening snap C. Mid-systolic click D. Paradoxically split S2

Explanations (u) A. Mitral stenosis is a diastolic, not a systolic murmur. (c) B. Mitral stenosis is characterized by a mid-diastolic opening snap. (u) C. Mid-systolic clicks are noted in mitral valve prolapse, not mitral stenosis. (u) D. Paradoxical splitting of S2 occurs in aortic stenosis not mitral stenosis.

68.

Scientific Concepts/Cardiology Which of the following is the most common cause for acute myocardial infarction? A. Occlusion caused by coronary microemboli B. Thrombus development at a site of vascular injury C. Congenital abnormalities D. Severe coronary artery spasm

Explanations (u) A. Coronary microemboli occlusion is a rare cause of acute myocardial infarction. (c) B. Acute myocardial infarction occurs when a coronary artery thrombus develops rapidly at a site of vascular injury. In most cases, infarction occurs when an atherosclerotic plaque fissures, ruptures, or ulcerates and when conditions favor thrombogenesis, so that a mural thrombus forms at the site of rupture and leads to coronary artery occlusion. (u) C. Congenital abnormalities are rare causes of acute MI. (u) D. Severe coronary artery spasm is more likely to result in Prinzmetal's angina rather than true infarction.

69.

Health Maintenance/Cardiology A 78 year-old male with history of coronary artery disease status post CABG and ischemic cardiomyopathy presents with complaint of progressive dyspnea and orthopnea. He also complains of lower extremity edema. The patient denies fever, chest pain, or cough. On physical examination, vital signs are BP 120/68, HR 75 and regular, RR 22, afebrile. You note the patient to have an S3 heart sound, jugular venous distention, and 2+ lower extremity edema. The patient is admitted and treated. Upon discharge from the hospital, the patient should be educated to monitor which of the following at home? A. Daily weights B. Daily spirometry C. Daily blood glucose D. Daily fat intake

Explanations (c) A. Home monitoring of daily weights can alert the health care provider to the early recognition of worsening heart failure. (u) B. Spirometry monitoring is important in a patient with asthma, not heart failure. (u) C. Daily blood glucose monitoring is important in a patient with diabetes, not heart failure. (u) D. Daily fat intake is important, but will not improve his heart failure management.

70.

Scientific Concepts/Cardiology Which of the following is the most common cause of arterial embolization? A. Rheumatic heart disease B. Myxoma C. Atrial fibrillation D. Venous thrombosis

Explanations (u) A. Rheumatic heart disease is a rare cause of embolization (u) B. Myxoma is a rare cause of embolization. (c) C. Atrial fibrillation is present in 60-70% of patients with arterial emboli and is associated with left atrial appendage thrombus. (u) D. Venous thrombosis may be a cause of embolization paradoxically, but is uncommon.

71.

Scientific Concepts/Cardiology The most common arrhythmia encountered in patients with mitral stenosis is A. atrial flutter. B. atrial fibrillation. C. paroxysmal atrial tachycardia. D. atrio-ventricular dissociation.

Explanations (u) A. See B for explanation. (c) B. Mitral stenosis leads to enlargement of the left atrium, which is the major predisposing risk factor for the development of atrial fibrillation. (u) C. See B for explanation. (u) D. See B for explanation.

72.

Clinical Therapeutics/Cardiology Long term use of which of the following drugs may cause a drug-induced lupustype eruption? A. prednisone B. tetracycline C. procainamide D. oral contraceptives

Explanations (u) A. Prednisone is not implicated in druginduced skin reactions. (u) B. Tetracycline and sulfonamides are known to cause a photosensitive rash on sun exposed areas of the skin. (c) C. Procainamide and hydralazine are the most common drugs that may cause a lupuslike eruption. (u) D. Oral contraceptives may induce erythema nodosum.

73.

Scientific Concepts/Cardiology Which of the following is a cause of high output heart failure? A. myocardial ischemia B. complete heart block C. aortic stenosis D. thyrotoxicosis

Explanations (u) A. Low output heart failure occurs secondary to ischemic heart disease, hypertension, dilated cardiomyopathy, valvular and pericardial disease, and arrhythmia. (u) B. See A for explanation. (u) C. See A for explanation. (c) D. High output heart failure occurs in patients with reduced systemic vascular resistance. Examples include: thyrotoxicosis, anemia, pregnancy, beriberi and Paget's disease. Patients with high output heart failure usually have normal pump function, but it is not adequate to meet the high metabolic demands.

74.

Diagnosis/Cardiology A 46 year-old male with no past medical history presents complaining of chest pain for four hours. The patient admits to feeling very poorly over the past two weeks with fever and upper respiratory symptoms. The patient denies shortness of breath or diaphoresis. On examination the patient appears fatigued. Vital signs reveal a BP of 130/80, HR 90 and regular, RR 14. The patient is afebrile. Labs reveal a Troponin I of 10.33 ug/L (0-0.4ug/L). Cardiac catheterization shows normal coronary arteries and an ejection fraction of 40% with global hypokinesis. Which of the following is the most likely diagnosis? A. myocarditis B. pericarditis C. hypertrophic cardiomyopathy D. coronary artery disease

Explanations (c) A. Myocarditis often occurs secondary to acute viral illness and causes cardiac dysfunction. Patients will commonly have a history of a recent febrile illness. Chest pain may mimic that of a myocardial infarction and Troponin I levels maybe elevated in onethird of patients. Contractile dysfunction is seen on catheterization and/or echocardiogram. (u) B. Pericarditis does not typically cause ventricular dysfunction and cardiac enzymes are usually normal. (u) C. Hypertrophic cardiomyopathy is associated with ventricular hypercontractility. (u) D. This patient had normal coronary arteries on cardiac catheterization, no signs of coronary artery disease.

75.

Clinical Therapeutics/Cardiology Which of the following antihypertensive agents is considered to have both alpha- and beta-blocker activities? A. carvedilol (Coreg) B. hydralazine (Apresoline) C. minoxidil (Loniten) D. spironolactone (Aldactone)

Explanations (c) A. Carvedilol has both alpha- and betablocker activities. (u) B. Hydralazine and minoxidil are considered vasodilators. (u) C. See B for explanation. (u) D. Spironolactone is a potassium-sparing diuretic.

76.

Diagnosis/Cardiology A 12 year-old boy presents to the office with pain in his legs with activity gradually becoming worse over the past month. He is unable to ride a bicycle with his friends due to the pain in his legs. Examination of the heart reveals an ejection click and accentuation of the second heart sound. Femoral pulses are weak and delayed compared to the brachial pulses. Blood pressure obtained in both arms is elevated. Chest x-ray reveals rib notching. Which of the following is the most likely diagnosis? A. abdominal aortic aneurysm B. pheochromocytoma C. coarctation of the aorta D. thoracic outlet syndrome

Explanations (u) A. Abdominal aortic aneurysm is usually asymptomatic until the patient has dissection or rupture. It is uncommon in a child. (u) B. Pheochromocytoma classically causes paroxysms of hypertension due to catecholamine release from the adrenal medulla, but does not cause variations in blood pressure in the upper and lower extremities. (c) C. Coarctation is a discrete or long segment of narrowing adjacent to the left subclavian artery. As a result of the coarctation, systemic collaterals develop. X-ray findings occur from the dilated and pulsatile intercostal arteries and the "3" is due to the coarctation site with proximal and distal dilations. (u) D. Thoracic outlet syndrome occurs when the brachial plexus, subclavian artery, or subclavian vein becomes compressed in the region of the thoracic outlet. It is the most common cause of acute arterial occlusion in the upper extremity of adults under 40 years old.

77.

Clinical Therapeutics/Cardiology According to the recent JNC VII guidelines, a 34 year-old male who has type 1 diabetes mellitus and hypertension should be started on which type of antihypertensive agent? A. beta-blocker B. loop diuretic C. ACE inhibitor D. thiazide diuretic

Explanations (u) A. Beta blockers could potentially be harmful in a patient with diabetes mellitus. Use a cardioselective beta- blocker to reduce the incidence of hypoglycemia. (u) B. See C for explanation. (c) C. ACE inhibitors are effective in young patients. They are capable of providing protection to the kidney especially in diabetes mellitus. (u) D. See C for explanation.

78.

Scientific Concepts/Cardiology A patient presents with moderate mitral stenosis. Which of the following complications is associated with an increased risk of systemic embolization in this patient? A. atrial fibrillation B. pulmonary hypertension C. increased left atrial pressure D. left ventricular dilatation

Explanations (c) A. 50-80% of patients with mitral stenosis will develop paroxysmal or chronic atrial fibrillation; 20-30% of patients with atrial fibrillation will have systemic embolization. (u) B. Pulmonary hypertension can occur in patients with severe mitral stenosis with symptoms of low cardiac output and right sided heart failure. Pulmonary hypertension does not cause systemic embolization. (u) C. Patients with mitral stenosis can have increased left atrial pressures relative to the left ventricular diastolic pressures; this does not usually cause systemic embolization. (u) D. Left ventricular dilatation is more common in aortic valve disease than mitral valve disease.

79.

Diagnostic Studies/Cardiology A 19 year-old female presents with complaint of palpitations. On examination you note the patient to have particularly long arms and fingers and a pectus excavatum. She has a history of joint dislocation and a recent ophthalmologic examination revealed ectopic lentis. Which of the following echocardiogram findings would be most consistent with this patient's physical features? A. right atrial enlargement B. aortic root dilation C. pulmonic stenosis D. ventricular septal defect

Explanations (u) A. Patients with Marfan's syndrome commonly have mitral valve prolapse and possibly aortic regurgitation. Right atrial enlargement, pulmonic stenosis and ventricular septal defect are not commonly seen. (c) B. This patient has the signs and symptoms consistent with Marfan's syndrome. Ectopia lentis, aortic root dilation and aortic dissection are major criteria for the diagnosis of the disease. (u) C. See A for explanation. (u) D. See A for explanation.

80.

Diagnosis/Cardiology A patient presents with chest pain. ECG done in the emergency department reveals ST segment elevation in leads II, III, and AVF. This is most consistent with a myocardial infarction in which of the following areas? A. anterior wall B. inferior wall C. posterior wall D. lateral wall

Explanations (u) A. Anterior wall myocardial infarction is characterized by ST segment elevation in 1 or more of the precordial (V1- V6) leads. (c) B. Inferior wall myocardial infarction is characterized by ST segment elevation in leads II, III, and AVF. (u) C. Posterior wall myocardial infarction is characterized by ST segment depression in leads V1-V3 and a large R wave in leads V1-V3. (u) D. Lateral wall myocardial infarction is characterized by ST segment elevation in leads I and AVL.

81.

Clinical Therapeutics/Cardiology Which of the following is an absolute contraindication to thrombolytic therapy in a patient with an acute ST segment elevation myocardial infarction? A. history of severe hypertension presently controlled B. current use of anticoagulation therapy C. previous hemorrhagic stroke D. active peptic ulcer disease

Explanations (u) A. See C for explanation. (u) B. See C for explanation. (c) C. Absolute contraindications to thrombolytic therapy include a previous hemorrhagic stroke, a stroke within one year, a known intracranial neoplasm, active internal bleeding, and a suspected aortic dissection. Severe, but controlled hypertension, use of anticoagulation, and active peptic ulcer disease are relative contraindications in which the risk/benefit ratio must be weighed in each patient. (u) D. See C for explanation.

82.

Health Maintenance/Cardiology A postmenopausal woman is at greatest risk of death from which of the following? A. stroke B. heart disease C. ovarian cancer D. breast cancer

Explanations (u) A. See B for explanation. (c) B. Although women tend to be concerned about dying from breast cancer, heart disease is the number one killer of postmenopausal women. (u) C. See B for explanation. (u) D. See B for explanation.

83.

Diagnosis/Cardiology A 46 year-old female is being evaluated for a new-onset hypertension that was discovered on screening at her workplace. The patient had several readings revealing systolic and diastolic hypertension. Patient is currently on no medications. Physical examination is unremarkable. A complete laboratory evaluation revealed hypokalemia as the only abnormality. Which of the following is the most likely diagnosis for this patient? A. pheochromocytoma B. renal artery stenosis C. coarctation of the aorta D. primary aldosteronism

Explanations (u) A. Pheochromocytoma will result in an increase in the production and release of catecholamines, which results in an increase in urinary metanephrines on testing. (u) B. Renal artery stenosis is identified by an abnormal radionuclide uptake on the affected kidney. (u) C. Coarctation of the aorta is identified by delayed and weakened femoral pulses along with a blood pressure in the lower extremities significantly lower than in the upper extremities. (c) D. Primary aldosteronism has an increased aldosterone secretion, which causes the retention of sodium and the loss of potassium. This should be the primary consideration for this patient.

84.

Clinical Intervention/Cardiology A 54 year-old female who has diabetes presents with rubor, absence of hair, and brittle nails of her left foot. She complains of leg pain that awakens her at night. Examination reveals a femoral bruit with diminished popliteal and pedal pulses on the left side. The most appropriate therapy would be A. vasodilator therapy. B. bypass surgery. C. exercise program. D. embolectomy.

Explanations (u) A. Vasodilator therapy is not indicated. (c) B. Bypass surgery is indicated in the presence of rest pain and provides relief of symptoms in 80 to 90% of patients. (u) C. While an exercise program is appropriate with claudication, rest pain is a surgical indication. (u) D. Embolectomy is used for acute arterial occlusion.

85.

Clinical Therapeutics/Cardiology Which electrolyte abnormality is associated with an increase in the risk for digoxin toxicity?] A. hypercalcemia B. hypokalemia C. hypermagnesemia D. hyponatremia

Explanations (u) A. See B for explanation. (c) B. Decreased concentration of potassium results in the increased activity of cardiac glycosides by increasing tissue binding and decreasing renal excretion of digoxin. Potassium loss is the only significant electrolyte abnormality that significantly affects digoxin metabolism. (u) C. See B for explanation. (u) D. See B for explanation.

86.

Health Maintenance/Cardiology A 56 year-old male, status post myocardial infarction, is noted to have left ventricular hypertrophy and an ejection fraction of 38%. Which of the following medications should be prescribed to prevent the development of heart failure symptoms? A. amlodipine (Norvasc) B. furosemide (Lasix) C. hydrochlorothiazide (HCTZ) D. lisinopril (Zestril)

Explanations (u) A. See D for explanation. (u) B. See D for explanation. (u) C. See D for explanation. (c) D. ACE inhibitors have been shown to markedly improve survival and are also recommended for prevention of symptoms in patients at risk for heart failure.

87.

Health Maintenance/Cardiology A 74 year-old patient presents with signs and symptoms of heart failure. EKG shows the patient to be in atrial fibrillation at a rate of 80 bpm. Blood pressure is 120/76. The patient denies complaint of palpitations, chest pain, or syncope. Which of the following is the most important long term therapy in this patient? A. verapamil (Calan) B. amiodarone (Cordarone) C. furosemide (Lasix) D. warfarin (Coumadin)

Explanations (u) A. Calcium channel blockers are utilized in rate control of atrial fibrillation. This patient's rate is controlled at 80bpm presently. (u) B. Antiarrhythmic therapy may be considered in patients with atrial fibrillation; however anticoagulation therapy must occur first. (u) C. Diuretics may be indicated in the acute treatment of heart failure; however they may not be needed long term. (c) D. Patients with atrial fibrillation have an increased risk for stroke, therefore these patients need anticoagulation with warfarin to an INR of 2.0-3.0.

88.

Diagnostic Studies/Cardiology Which of the following ECG findings is consistent with hyperkalemia? A. prolonged QT interval B. delta wave C. peaked T waves D. prominent U waves

Explanations (u) A. Prolonged QT interval is seen in hypocalcemia. (u) B. Delta wave is a sign of ventricular preexcitation seen in Wolf-ParkinsonWhite (WPW) Syndrome. (c) C. Narrowing and peaking of T waves are the beginning EKG changes associated with hyperkalemia. (u) D. Prominent U waves are a sign of prolonged ventricular repolarization seen in hypokalemia.

89.

History & Physical/Cardiology A 58 year-old male presents with chest pain. Vital signs include blood pressure of 210/175, pulse 80, RR 20. Which of the following would you expect to find on physical examination? A. papilledema B. carotid bruit C. diastolic murmur D. absent peripheral pulses

Explanations (c) A. Malignant hypertension is characterized by marked blood pressure elevation with papilledema, often with encephalopathy or nephropathy. (u) B. Carotid bruits are associated with carotid artery stenosis. (u) C. Diastolic murmurs are associated with valvular heart disease such as aortic regurgitation and mitral stenosis. (u) D. Peripheral pulses are absent in acute arterial occlusion or severe peripheral arterial disease.

90.

Clinical Therapeutics/Cardiology A 55 year-old diabetic female presents for a 3 month blood pressure followup. At the last visit the BP was 160/90 for the third consecutive visit. She was placed on an ACE inhibitor and educated regarding lifestyle modifications. At today's visit the patient complains of persistent annoying dry cough that has been going on since the last visit. BP today is 120/70. What is the best recommendation to control her BP? A. add a diuretic B. stop the ACE inhibitor and continue lifestyle modifications C. switch patient to an Angiotensin II Receptor Blocker (ARB) D. do nothing and recheck BP in 3 months

Explanations (u) A. This patient's blood pressure is controlled; there is no indication at this time to add an additional drug. (u) B. This patient's chronic dry cough is likely secondary to the ACE inhibitor, the medication should be stopped, however the patient needs something for blood pressure control. (c) C. This patient's chronic dry cough is likely secondary to the ACE inhibitor, the medication should be stopped. Angiotensin II Receptor Blockers (ARBs) are similar to ACE inhibitors for BP control, but do not cause cough. (u) D. This patient's chronic dry cough is likely secondary to the ACE inhibitor, the medication should be stopped to encourage compliance.

91.

Diagnosis/Cardiology A newborn is seen for an initial two week visit. Physical examination reveals a thrill and a continuous machinery murmur in the left second intercostal space. Which of the following is the most likely diagnosis? A. patent ductus arteriosus B. ventricular septal defect C. tetralogy of Fallot D. coarctation of the aorta

Explanations (c) A. Patent ductus arteriosus is characterized by a classic harsh, machinery-like murmur that is continuous through systole and diastole. This is heard best at the left second interspace and is commonly associated with a thrill. (u) B. Ventricular septal defect is characterized by a holosystolic murmur at the lower left sternal border. (u) C. Tetralogy of Fallot is characterized by a systolic thrill at the left sternal border with a systolic ejection murmur that may or may not have an associated systolic click. (u) D. Coarctation of the aorta is associated with a systolic ejection click or a short systolic murmur at the left sternal border.

92.

History & Physical/Cardiology A patient had an acute inferior, transmural myocardial infarction 4 days ago. A new murmur raises the suspicion of mitral regurgitation due to papillary muscle rupture. Which of the following murmur descriptions describes this condition? A. A grade III/VI diastolic murmur heard best at the apex without radiation. B. A grade IV/VI systolic ejection murmur heard best at the base with radiation to the left clavicle. C. A grade II/VI systolic murmur heard best at the apex preceded by a click and without radiation. D. A grade IV/VI systolic murmur heard best at the apex with radiation to the left axilla.

Explanations (u) A. This is a classic description of mitral stenosis. (u) B. This is a classic description for pulmonic stenosis. (u) C. This is a classic description for mitral valve prolapse. (c) D. This is a classic description of mitral regurgitation. The papillary muscle rupture is a complication of an acute inferior transmural myocardial infarction, and results in a failure of the mitral valve leaflets to close. The direction of regurgitant flow of blood is toward the left axilla.

93.

Clinical Intervention/Cardiology A 58 year-old male who is otherwise healthy presents with chest pain and is found to have left main coronary artery stenosis of 75%. The most important aspect of his management now is A. daily aspirin to prevent MI. B. nitrate therapy for the angina. C. aggressive risk factor reduction. D. referral for coronary artery revascularization.

Explanations (u) A. See D for explanation. (u) B. See D for explanation. (u) C. See D for explanation. (c) D. Although medical therapy is important, revascularization is indicated when stenosis of the left main coronary artery is greater than 50%.

94.

Diagnostic Studies/Cardiology A 17 year-old woman presents to the office with recurrent episodes of palpitations and near syncope. Initial ECG was normal. She is concerned about these episodes since they can occur at any time. Which of the following is the most appropriate step to pursue in her evaluation? A. cardiac catheterization B. tilt table testing C. echocardiogram D. Holter monitoring

Explanations (u) A. A cardiac catheterization will not be useful since the patient is at low risk for actual coronary artery disease. (u) B. Tilt table testing is useful only in trying to determine vasodepressor syncope that is related to position. (u) C. An echocardiogram shows valves and left ventricle function, not pathways of conduction. (c) D. Holter monitoring will identify the heart rhythm; an event recorder may also be useful in this setting if the Holter monitor is not diagnostic.

95.

Health Maintenance/Cardiology A 37 year-old female with history of Turner's syndrome and coarctation of the aorta repaired at the age of 3 presents for routine examination. The patient is without complaints of chest pain, dyspnea, palpitations, or syncope. On examination vitals signs reveal a BP of 130/76, HR 70, regular, RR 16. On cardiac examination you note a grade II/VI systolic ejection murmur at the left sternal border and a grade III/VI blowing diastolic murmur. Which of the following does this patient require? A. antibiotic prophylaxis B. beta blocker therapy C. chest CT D. exercise stress test

Explanations (c) A. This patient has a history of congenital heart disease and presently has a murmur consistent with aortic regurgitation. This patient requires antibiotic prophylaxis against infective endocarditis. (u) B. Beta blocker therapy may increase the amount of regurgitation because of increased diastolic time and is not indicated in this patient. (u) C. This patient should undergo serial echocardiograms, chest CT will not give information regarding any changes in the aortic regurgitation or ejection fraction. (u) D. This patient is without any complaints; exercise stress test is not indicated.

96.

Diagnostic Studies/Cardiology A 60 year-old male with history of hypertension and hyperlipidemia presents with intermittent chest heaviness for one month. The patient states he has had occasional heaviness in his chest while walking on his treadmill at home or shoveling snow. He also admits to mild dyspnea on exertion. His symptoms are relieved with 2-3 minutes of rest. He denies lightheadedness, syncope, orthopnea or lower extremity edema. Vitals reveal a BP of 130/90, HR 70, regular, RR 14. Cardiac examination revealed a normal S1 and S2, without murmur or rub. Lungs were clear to auscultation. Extremities are without edema. EKG reveals no acute change and cardiac enzymes are negative. Which of the following is the most appropriate next diagnostic study? A. cardiac catheterization B. nuclear exercise stress test C. helical (spiral) CT D. transthoracic Echocardiogram

Explanations (u) A. Coronary angiography is indicated in patients with classic stable angina who are severely symptomatic despite medical therapy and are being considered for percutaneous intervention (PCI), patients with troublesome symptoms that are difficult to diagnose, angina symptoms in a patient who has survived sudden cardiac death event, patients with ischemia on noninvasive testings, a stress test is a better initial diagnostic study for this patient. (c) B. This patient has signs and symptoms of classic angina; nuclear stress testing is the most useful noninvasive procedure for diagnosis of ischemic heart disease and evaluation of angina in this patient. (u) C. Helical CT is used in the diagnosis of pulmonary embolism, not in the evaluation of angina. (u) D. Echocardiogram is used in the evaluation of valvular heart disease not in the evaluation of suspected myocardial ischemia.

97.

History & Physical/Cardiology A 28 year-old patient presents with complaint of chest pain for two days. The patient describes the pain as constant and sharp. It is worse with lying down, better with sitting up and leaning forward. Vital signs are BP 120/80, HR 80, regular, RR 14 and Temperature 100.1 degrees F. Which of the following would you expect to find on physical examination? A. lower extremity edema B. carotid bruit C. pericardial friction rub D. splinter hemorrhages

Explanations (u) A. Lower extremity edema is seen with heart failure or venous insufficiency, not pericarditis. (u) B. Carotid bruits are associated with carotid artery stenosis, not pericarditis. (c) C. This patient has signs and symptoms of pericarditis. A pericardial friction rub is characteristic of acute pericarditis. (u) D. Subungual (splinter) hemorrhages are characteristic of infective endocarditis, not pericarditis.

98.

History & Physical/Cardiology A 22 year-old male received a stab wound in the chest an hour ago. The diagnosis of pericardial tamponade is strongly supported by the presence of A. pulmonary edema. B. wide pulse pressure. C. distended neck veins. D. an early diastolic murmur.

Explanations (u) A. Pulmonary edema may result with low output states as seen with myocardial contusions, but it is not strongly suggestive of tamponade. (u) B. Wide pulse pressure is seen in conditions of high stroke volume such as aortic insufficiency or hyperthyroidism. Narrow pulse pressure is seen with cardiac tamponade. (c) C. Cardiac tamponade will manifest with distended neck veins and cold clammy skin. (u) D. The onset of diastolic murmur is suggestive of valvular disease, not tamponade.

99.

Clinical Intervention/Cardiology A 45 year-old female presents with complaint of lower extremity discomfort. The patient admits to dull aching of the left lower extremity. The discomfort is worse after standing for long periods of time. Examination reveals dilated, tortuous and elongated veins on the medial aspect of the left leg. Pedal pulses are +2/4 bilaterally. There are no skin changes or lower extremity edema noted. Which of the following is the most appropriate initial treatment of choice in this patient? A. heparin B. compression stockings C. furosemide (Lasix) D. thrombectomy

Explanations (u) A. Heparin is used in the treatment of deep vein thrombosis not varicose veins. (c) B. This patient has signs and symptoms of varicose veins. Initial treatment with compression stockings may prolong or avoid the need for surgery. (u) C. This patient has no signs of edema or venous insufficiency requiring diuretic therapy. (u) D. Thrombectomy is indicated in a patient with an arterial thrombus, this patient has intact pulses and no pallor.

100.

Diagnosis/Cardiology A 55 year-old male presents with complaint of sudden ripping chest pain that radiates into the abdomen. On examination the patient is found to have diminished peripheral pulses and a diastolic murmur. EKG reveals left ventricular hypertrophy. Which of the following is the most likely diagnosis? A. acute myocardial infarction B. pulmonary embolism C. acute pericarditis D. aortic dissection

Explanations (u) A. Pain associated with a myocardial infarction is commonly a retrosternal pressure, squeezing, or heaviness. ST segment elevation on EKG would be expected. (u) B. A pulmonary embolism is associated with retrosternal pain; however chest pain is not always present. Patients more commonly will have a sudden onset of dyspnea. PE is not usually associated with a diastolic murmur or diminished pulses. (u) C. Acute pericarditis is characterized by sharp, knifelike pain that is worse with lying supine and better with sitting up and leaning forward. One would expect to find a pericardial friction rub on auscultation. (c) D. Aortic dissection is characterized by a ripping or tearing type pain with radiation to the neck, back or abdomen. Left ventricular hypertrophy is often seen on EKG secondary to longstanding hypertension. A diastolic murmur is often present secondary to aortic insufficiency.

101.

History & Physical/Cardiology A 78 year-old patient who is in acute distress with near-syncope and lightheadedness is being examined. The patient's pulse is 40/min and blood pressure is 90/56 mm Hg. Examination of the patient at 45 degrees of elevation reveals cannon "a" waves. Which of the following is the most likely explanation for these abnormal waves? A. atrioventricular dissociation B. aortic stenosis C. systolic hypertension D. left ventricular hypertrophy

Explanations (c) A. The patient is in a third-degree heart block with the atria contracting against a closed atrioventricular valve, which would be the scenario in a patient who has an escape rate of 40. Elderly patients are at risk for heart conduction problems such as complete heart block. (u) B. Left ventricular hypertrophy, systolic hypertension, and aortic stenosis are not causes of "a" waves. (u) C. See B for explanation. (u) D. See B for explanation.

102.

Scientific Concepts/Cardiology Which of the following occurs as a result of pulmonary hypertension? A. left atrial enlargement B. aortic stenosis C. coronary artery spasm D. right ventricular enlargement

Explanations (u) A. See D for explanation. (u) B. See D for explanation. (u) C. See D for explanation. (c) D. In pulmonary hypertension increased resistance within pulmonary circulation causes the right ventricle to work harder and eventually enlarge in response. Other changes that may occur are right atrial enlargement, decreased left ventricular cavity size, and tricuspid regurgitation.

103.

Diagnostic Studies/Cardiology A 56 year-old male presents to the office with a history of abdominal aortic aneurysm. He was told that he will need ongoing evaluation to assess whether the aneurysm is expanding. What is the recommended study to utilize in this situation? A. plain film of the abdomen B. serial abdominal exam C. ultrasound of the abdomen D. angiography of the abdominal aorta

Explanations (u) A. Although some abdominal aortic aneurysms are calcified, abdominal radiography may demonstrate the calcified outline of the aneurysm. However, about 25% of aneurysms are not calcified and cannot be visualized by plain x-ray. (u) B. Serial abdominal exams are not sensitive in detecting progression of abdominal aortic aneurysms. (c) C. An abdominal ultrasound can delineate the transverse and longitudinal dimensions of an abdominal aortic aneurysm and may detect mural thrombus. Abdominal ultrasound is best used to screen patients at risk for the development of this condition. (u) D. Contrast aortography is used commonly for the evaluation of patients with aneurysms before surgery, but it has no role in the serial assessment of patients being followed on a chronic basis.

104.

Diagnosis/Cardiology A 55 year-old morbidly obese male is seen in the office for routine examination. He has a history of pulmonary hypertension and cor pulmonale. Examination reveals a visible jugular venous pulse and a systolic flow murmur on the right side of the sternum. Which of the following is the most likely diagnosis? A. mitral insufficiency B. tricuspid insufficiency C. hepatic vein thrombosis D. aneurysm of the thoracic aorta

Explanations (u) A. Mitral insufficiency results in the accumulation of blood primarily in the pulmonary system and not the right side of the heart. (c) B. Tricuspid insufficiency will result in blood being put back into the right side of the body with increased jugular pulsation in the neck, along with a palpable venous pulse in the liver. (u) C. Hepatic vein thrombosis or Budd-Chiari syndrome is associated with cirrhosis and liver clotting abnormalities and is not due to right-sided heart failure. (u) D. Thoracic aorta aneurysm results in a widened mediastinum that is fairly asymptomatic until it results in rupture or dissection. These are typically found as incidental findings unless they are symptomatic from dissection or rupture, which causes severe chest pain or a severe tearing sensation in the chest.

105.

Clinical Intervention/Cardiology A 68 year-old patient presents after a syncopal episode. The patient has a history of coronary artery disease and ischemic cardiomyopathy. Echocardiogram shows an ejection fraction of 20%. Electrophysiology study reveals inducible sustained ventricular tachycardia from the left ventricle. Which of the following is the most appropriate therapy in this patient? A. implantable defibrillator B. metoprolol (Lopressor) C. radiofrequency ablation D. warfarin (Coumadin)

Explanations (c) A. Patients with symptomatic ventricular tachycardia (VT) or sustained VT and left ventricular dysfunction are at increased risk for sudden cardiac death. An implantable defibrillator is the treatment of choice. (u) B. Beta blockers are used in patients with nonsustained VT and normal ventricular function. They may be used as an adjunct to, but not in place of, implantable defibrillator therapy in patients with symptomatic VT or sustained VT. (u) C. Radiofrequency ablation is indicated in patients with outflow tract or fascicular tachycardia, not left sided VT. (u) D. Anticoagulation therapy is indicated in patients with atrial fibrillation not VT.

106.

Scientific Concepts/Cardiology Which of the following is the most common cause of infective endocarditis in an IV drug abuser? A. Haemophilus parainfluenza B. Enterococci C. Staphylococcus aureus D. Viridans streptococci

Explanations (u) A. See C for explanation. (u) B. See C for explanation. (c) C. S. aureus accounts for more than 60% of all cases of endocarditis in IV drug abusers. (u) D. See C for explanation.

107.

History & Physical/Cardiology Which of the following is an expected finding in a patient with a diagnosis of an arterial embolism? A. lower extremity edema B. stasis dermatitis C. palpable cord D. pulselessness

Explanations (u) A. Lower extremity edema is commonly associated with venous insufficiency, not arterial embolism. (u) B. Stasis dermatitis is commonly seen in patients with venous insufficiency, not arterial embolism. (u) C. A palpable cord is commonly found in a patient with a DVT, not arterial embolism. (c) D. Pulselessness is a sign of acute ischemia secondary to arterial embolism.

108.

Scientific Concepts/Cardiology Which of the following factors in patients with chronic venous insufficiency predisposes them to development of skin ulcers? A. Increased intravascular oncotic pressure B. Leakage of fibrinogen and growth factors into the interstitial space C. Decreased capillary leakage D. Inherited deficiency of protein C

Explanations (u) A. Decreased intravascular oncotic pressure can cause swelling (c) B. Leakage of fibrinogen and growth factors into the interstitial space, leukocyte aggregation and activation, and obliteration of the cutaneous lymphatic network can predispose a patient to skin ulcers (u) C. Increased capillary leakage causes venous insufficiency. (u) D. Inherited deficiency of protein C predisposes patients to thrombosis.

109.

Diagnosis/Cardiology A newborn is being evaluated for perioral cyanosis while feeding associated with sweating. Vital signs are rectal temperature, 37.8 degrees C (100 degrees F), blood pressure 80/45 mmHg, pulse 180/min, and respirations 40/min. A grade 3/6 harsh systolic ejection murmur with a single loud S2 is heard at the left upper sternal border. Electrocardiogram (ECG) shows right ventricular hypertrophy with right axis deviation. Chest x-ray shows a boot- shaped heart and decreased pulmonary vascular markings. Which of the following is the most likely diagnosis? A. Atrial septal defect B. Total anomalous pulmonary venous return C. Coarctation of the aorta D. Tetralogy of Fallot

Explanations (u) A. Although the murmur may be consistent with an ASD with pulmonary hypertension the chest x-ray would not show decreased pulmonary vascular markings. With a large left to right shunt large pulmonary arteries and increased vascularity would be seen. (u) B. The murmur for TAPVR is a soft systolic murmur at the left upper sternal border with a split S2 in addition to a short mid-diastolic murmur at the low left sternal border. (u) C. Cyanosis is usually not the presenting sign for coarctation of the aorta. Infants may present with heart failure, ECG will show evidence of LVH. (c) D. This is a common presentation for tetralogy of fallot.

110.

Clinical Therapeutics/Cardiology A hospitalized patient is found with confirmed pulseless ventricular tachycardia. IV access is obtained following the second shock given. Which of the following medications is to be administered immediately? A. Amiodarone B. Magnesium C. Atropine D. Epinephrine

Explanations (u) A. Antiarrhythmics are given after the third shock and epinephrine has been administered. (u) B. Magnesium is useful for torsades de pointes. (u) C. Atropine may be used for asystole or a slow pulseless electrical activity (PEA) rate. (c) D. Epinephrine should be given as soon as IV access is obtained before or after the second shock.

111.

Diagnosis/Cardiology An electrocardiogram (ECG) shows a sinus rhythm with varying T wave heights, axis changes every other beat and a wandering baseline. Which of the following is most likely the diagnosis? A. Artifact B. Digoxin toxicity C. Pericardial effusion D. Poor lead placement

Explanations (u) A. Artifact could show a wandering baseline, but not the distinct axis changes. (u) B. Digoxin toxicity can cause bidirectional tachycardia, but not electrical alternans. (c) C. This ECG pattern best represents pericardial effusion due to a swinging heart in fluid and is known as electrical alternans. (u) D. Poor lead placement would show different patterns per the leads.

112.

Clinical Therapeutics/Cardiology A 25 year-old female presents to the emergency department after an episode of substernal chest pain with radiation to the middle of her back that came on suddenly and lasted for about four minutes this morning while in bed. She denies previous episodes. Examination is unremarkable, but she appears jittery. Toxicology screen is positive for cocaine. Which of the following medications is contraindicated in this patient? A. Lorazepam (Ativan) B. Diltiazem (Cardizem) C. Nitroglycerin (Nitrostat) D. Propanolol (Inderal)

Explanations (u) A. Lorazepam is not contraindicated and can help with agitation, psychosis or seizures. (u) B. Diltiazem is not contraindicated but does not have a definitive role in treating cocaine toxicity. (u) C. Nitroglycerin is not contraindicated but does not have a definitive role in treating cocaine toxicity. (c) D. Pure Beta blockers, such as propranol, can cause a paradoxical hypertension because of unopposed alphaadrenergic effects.

113.

Clinical Intervention/Cardiology A 56 year-old female four days post myocardial infarction presents with a new murmur. On examination the murmur is a grade 3/6 pansystolic murmur radiating to the axilla. She is dyspenic at rest and has rales throughout all her lung fields. Blood pressure is 108/68 mmHg, pulse 70 bpm. Which of the following would be the definitive clinical intervention? A. Intra-aortic balloon counterpulsation B. Mitral valve replacement C. Coronary artery bypass surgery D. Immediate fluid bolus

Explanations (u) A. Although part of the primary treatment to reduce mitral regurgitation, it is not definitive. (c) B. MVR is the definitive intervention to correct MR caused by papillary muscle rupture. (u) C. CABG may be necessary if significant blockage is found, but it will not correct the mitral regurgitation. (u) D. A fluid bolus is indicated if the patient is hypotensive.

114.

Clinical Therapeutics/Cardiology A 16 year-old male with a history of tetralogy of Fallot presents to clinic for a follow-up visit status post replacement of his right ventricle to pulmonary artery conduit. He has complaints of chest pain with inspiration, fever and general malaise. Cardiac examination reveals a rub with muffled heart sounds. Labs show an elevated erythrocyte sedimentation rate (ESR) and leukocytosis. Which of the following is the most effective treatment? A. Acetaminophen/oxycodone B. Azithromycin C. Indomethacin D. Furosemide

Explanations (u) A. See C for explanation. (u) B. See C for explanation. (c) C. Indomethacin is suitable for controlling pain in Dressler's syndrome. ASA is preferred. Narcotics, diuretics or antibiotics are not recommended. (u) D. See C for explanation.

115.

Diagnostic Studies/Cardiology A 72 year-old male presents to the emergency department with crushing chest pain, dyspnea and palpitations for 2 hours in duration. Enzymes are pending and he has been given aspirin and sublingual nitroglycerin. He is rushed to the catheterization lab where they find a totally occluded distal right coronary artery. Which of the following electrocardiogram (ECG) findings supports the diagnosis? A. Q waves in leads I, aVL, V5-V6 B. ST segment elevation in leads II, III, aVF C. Hyperacute T waves in leads I, aVL D. Flipped T waves with repolarization changes in leads V1-V4

Explanations (u) A. Q waves in leads I, aVL, V5-V6 represent infarction involving the circumflex artery. (c) B. ST segment elevation in leads II, III, aVF, represents an acute process in the right coronary artery. (u) C. Hyperacute T waves in leads I, aVL can represent the initial changes of an infarction involving the circumflex artery. (u) D. Flipped T waves with repolarization changes in leads V1-V4 can represent early stages of infarction involving the left anterior descending artery.

116.

Health Maintenance/Cardiology Which of the following population groups represent the greatest risk for developing primary hypertension? A. White non-Hispanic B. Hispanic C. Mediterranean D. Black non-Hispanic

Explanations (u) A. White non-Hispanic adults have a low risk of hypertension compared to Hispanic and Black nonHispanics. (u) B. Hispanic adults are lower risk of hypertension than Black non-Hispanic, but not compared to White non- Hispanic. (u) C. Mediterranean adults have a lower risk of hypertension than Black non-Hispanics. (c) D. Black non-Hispanic adults have the highest risk of hypertension.

117.

Diagnosis/Cardiology A 60 year-old male with hypertension is brought to the emergency department 30 minutes after the sudden onset of severe chest pain that radiates to his back and arms. His blood pressure is 180/80 mmHg in his left arm; no blood pressure reading can be obtained from the right arm. ECG shows sinus tachycardia with left ventricular hypertrophy. A high pitched decrescendo diastolic murmur is heard along the left mid-sternal border. Which of the following is the most likely diagnosis? A. Acute myocardial infarction B. Aortic dissection C. Pulmonary embolism D. Right subclavian arterial embolus

Explanations (u) A. Although included as part of the differential the presentation is not consistent with AMI. ECG may show ST changes and a murmur of mitral regurgitation may be present with papillary muscle rupture. (c) B. This is a classic presentation for aortic dissection. (u) C. Patients will also present with shortness of breath, feelings of impending doom and chest pain that varies with respirations. (u) D. Arterial embolus will present with symptoms related to the location of the occlusion. Pain and paresthesias are usually the earliest symptoms.

118.

History & Physical/Cardiology Which of the following conditions is most suggestive of an abdominal aortic aneurysm? A. Abdominal mass B. Hypertension C. Chest pain D. Syncope

Explanations (c) A. An abdominal aortic aneurysm presents with a pulsatile upper abdominal mass. (u) B. Hypertension is not suggestive of an abdominal aortic aneurysm. (u) C. Abdominal aortic aneurysm presents with midabdominal or lower back pain. (u) D. Syncope is not common in abdominal aortic aneurysm, unless it ruptures.

119.

Health Maintenance/Cardiology Who is most likely to require subacute bacterial endocarditis (SBE) prophylaxis prior to a dental procedure? A. 22 year-old female with mitral valve prolapse B. 36 year-old male with a bio-prosthesic mitral valve C. 45 year-old female with an ASD closure 8 months ago with no residual defect D. 15 year-old male with a bicuspid aortic valve

Explanations (u) A. See B for explanation. (c) B. The AHA recommends that patients with prosthetic heart valves receive antibiotic prophylaxis. As should cardiac transplant recipients with valve disease, unrepaired cyanotic CHD, repaired CHD with prosthetic material or device during the first six months of the procedure and repaired CHD with residual defects at site of patch or prosthetic device. (u) C. See B for explanation. (u) D. See B for explanation.

120.

Scientific Concepts/Cardiology When evaluating jugular venous pulsations a prominent a wave represents which of the following? A. Atrial contraction against a closed tricuspid valve B. Rapid filling of the right atrium C. Tricuspid regurgitation D. Poor left ventricle compliance

Explanations (c) A. The a wave corresponds to right atrial contraction. (u) B. See A for explanation. (u) C. See A for explanation. (u) D. See A for explanation.

121.

History & Physical/Cardiology An 18 year-old male high school basketball player comes to clinic for a routine physical exam. His height is 193 cm (76 in.); arm span is 201 cm (79 in.). He has long fingers and toes. Blood pressure is 146/62 mmHg and pulse is 64/min. Which of the following exam findings is most consistent with the diagnosis? A. Grade 2/6 high-frequency diastolic murmur at the third right intercostal space B. Grade 2/6 systolic ejection murmur at the second left intercostal space with a fixed widely split S2 C. Grade 2/6 continuous murmur heard best at the high left sternal border D. Grade 2/6 systolic murmur at the fourth left intercostal space that decreases with squatting

Explanations (c) A. This murmur is most consistent with aortic regurgitation which can be present in patients with Marfans syndrome and a dilated aortic root. (u) B. This murmur is most consistent with an atrial septal defect. (u) C. This murmur is most consistent with a patent ductus arteriosus and unlikely in this age patient. (u) D. This murmur is most consistent with hypertrophic obstructive cardiomyopathy.

122.

Scientific Concepts/Cardiology Which of the following is the most likely initial effect on the left ventricle from aortic stenosis? A. Dilitation of the ventricle with diastolic dysfunction B. Wall stiffness due to ischemia from decreased coronary blood flow C. Paradoxical wall motion abnormalities due to increased preload D. Concentric hypertrophy with preserved function

Explanations (u) A. Dilitation of the ventricle is a later finding. (u) B. This is not an effect from aortic stenosis but coronary artery blockage. (u) C. Preload is the end-diastolic pressure or volume within the ventricle, ischemic heart disease or infarct would cause paradoxical wall motion abnormalities due to the increased preload. (c) D. Hypertrophy would be the initial changes of the left ventricle as a response to the increased pressure.

123.

Diagnosis/Cardiology A 65 year-old female who recently had an anterior MI returns to clinic for follow-up six weeks after. She has no chest pain, but reports decreased exercise tolerance. Electrocardiogram (ECG) shows persistent ST elevation in leads V2V4. Which of the following is the most likely diagnosis? A. Right ventricular infarction B. Re-occlusion of the right coronary artery C. Pericarditis D. Ventricular aneurysm

Explanations (u) A. RV infarction is present in one-third of patients who had an inferior wall MI and typically show ST elevation in V4 along with the inferior leads. (u) B. Occlusion of the right coronary artery would show ST elevation in the inferior ECG leads. (u) C. Pericarditis shows ST elevation in multiple leads with a history consistent of a viral illness or fever. (c) D. Persistent ST elevation in the leads where a previous or recent anterior MI occurred is most likely due to ventricular aneurysm.

124.

Diagnostic Studies/Cardiology A 15 year-old male is brought to the emergency department 1 hour after an episode of syncope while running in a 400-meter race. He had a similar episode 2 years ago. His mother and maternal first cousin died suddenly at the ages of 32 and 17 years, respectively. Examination shows abrasions of the face, hands, and knees. Neurologic examination shows no abnormalities. Which of the following is the most appropriate initial test? A. Tilt table test B. CT scan of the head C. Electrocardiogram D. Electroencephalography

Explanations (u) A. Tilt table testing should be performed after structural heart disease has been ruled out. (u) B. CT scanning of the head would not be the initial test of choice. (c) C. ECG is the most appropriate to rule out any underlying rhythm abnormalities, Holter or event monitoring would also be included. (u) D. EEG testing routinely would not be helpful as an initial test in this patient.

125.

Clinical Therapeutics/Cardiology What is the optimal INR for a patient with a mechanical mitral valve prosthesis on warfarin (Coumadin)? A. 1.5-2.5 B. 2.0-3.0 C. 2.5-3.5 D. 3.5-4.5

Explanations (u) A. See C for explanation. (u) B. See C for explanation. (c) C. Patients with mechanical MV prostheses should maintain an INR between 2.5-3.5. Aortic mechanical valves can be maintained at an INR of 2.0-3.0. (u) D. See C for explanation.

126.

Clinical Intervention/Cardiology Which of the following can optimize quality of life and is an definitive treatment for a patient with refractory heart failure? A. Ventricular assist device B. Intra-aortic balloon counterpulsation C. Cardiac transplantation D. Partial resection of the left ventricle

Explanations (u) A. Ventricular assist devices can help to provide temporary circulatory support to those awaiting transplantation. (u) B. IABC is useful in cardiogenic shock, but does not improve quality of life. (c) C. Cardiac transplantation is effective, with survival rates of 8090% in 1 year, 60-70% over 5 years. It does improve quality of life despite the immunosuppression medications. (u) D. Partial resection of the left ventricle is still experimental and has not shown to have successful results.

127.

History & Physical/Cardiology Which of the following physical exam findings suggests worsening or severe aortic stenosis? A. An ejection click preceding the murmur B. A split S2 with respiration variation C. Grade 2/6 systolic murmur radiating to the carotids D. Palpable thrill over the right second intercostal space

Explanations (u) A. This finding is typical in mild to moderate AS. (u) B. This can be a normal finding. (u) C. This is the typical murmur for aortic stenosis, but does not suggest the degree of severity by the grade. (c) D. A palpable thrill or LV heave with associated murmur suggests severe AS.

128.

Scientific Concepts/Cardiology Perfusion of the coronary arteries occurs primarily during which of the following? A. Systole B. Diastole C. Afterload D. Preload

Explanations (u) A. See B for explanation. (c) B. Coronary artery perfusion occurs primarily during diastole. (u) C. See B for explanation. (u) D. See B for explanation.

129.

Clinical Therapeutics/Cardiology What is the mechanism of action for aspirin? A. Inhibits platelet aggregation by blocking cyclooxygenase-1 activity B. Exerts antiplatelet effects by blocking the platelet membrane adenosine diphosphate receptors C. Inhibits the platelet membrane glycoprotein IIb/IIIa receptor D. Converts plasminogen to the natural fibrinolytic agent plasmin

Explanations (c) A. Aspirin inhibits platelet aggregation by blocking cyclooxygenase-1 activity. (u) B. This the mechanism of action for ADP antagonists such as clopidogrel or ticlopidine. (u) C. This is the mechanism of action for glycoprotein IIb/IIIa receptors such as abciximab (ReoPro). (u) D. This is the mechanism of action for tissue plasminogen activators (tPA).

130.

History & Physical/Cardiology A 56 year-old male with a 30 pack-year smoking history presents with substernal chest pain. The pain is described as a pressure that radiates to his jaw. The pain has lasted consistently for 30 minutes with variable relief. His current medications include atorvastatin (Lipitor) and glyburide (Micronase). Which of the following aggravating or relieving factors about the pain would support the diagnosis? A. Precipitated by exercise and relieved with rest B. Unrelieved by nitroglycerin C. Aggravated by deep breaths D. Relieved with food

Explanations (u) A. Precipitated by exercise and relief with rest is consistent with angina. (c) B. Patient is having a myocardial infarction which is unrelieved by rest or nitroglycerin. (u) C. Aggravated by deep breaths suggests pericarditis. (u) D. Relieved with food suggests peptic ulcer disease.

131.

Health Maintenance/Cardiology A 26 year-old female presents to clinic with a left arm that is swollen and non-tender with bluish discoloration along the upper arm and forearm. She is status post pacemaker insertion in the left upper chest for third degree heart block, one week ago. Pulses are present and the arm is warm, but not red. The pacemaker incision is healing well despite a hematoma and tenderness at the site. Which of the following statements would be appropriate patient education about this condition? A. Reassurance that the discoloration is an expected finding. B. Apply cold compresses to the site of the hematoma. C. Elevation of the involved extremity will increase the swelling. D. Aspirin should be taken to help manage pain.

Explanations (c) A. This is indicative of migratory ecchymosis and expected after insertion of a pacemaker. (u) B. Warm compresses will help to decrease the hematoma. (u) C. Elevation will help to decrease the swelling. (u) D. ASA is not the pain medicine of choice in a patient with a hematoma due to its non-clotting properties.

132.

Clinical Intervention/Cardiology A 66 year-old female with a history of coronary artery disease presents with a new onset of dizziness and fatigue for two weeks. She recalls nearly passing out on one occasion. Examination is unremarkable except for bradycardia. Electrocardiogram (ECG) reveals a heart rate of 50 with a normal PR interval followed by a normal QRS. There are several non-conducting P waves and no lengthening of the PR interval. Which of the following interventions is the therapy of choice? A. Permanent pacemaker B. Radio-frequency ablation C. Maze procedure D. Automatic Implantable Cardioverter Defibrillator

Explanations (c) A. This patient has symptomatic second degree type II heart block and requires a pacemaker. (u) B. Radio-frequency ablation is useful for supraventricular tachyarrhythmias. (u) C. Maze procedure is a surgical procedure for decreasing the incidence of atrial fibrillation by creating cuts simulating a pathway in the atriums. (u) D. Automatic Implantable Cardioverter Defibrillator (AICD) is used in patients at risk for sudden death.

133.

Diagnostic Studies/Cardiology Pulmonary capillary wedge pressure indirectly measures which of the following? A. Right ventricular end-diastolic pressure B. Right atrial filling pressure C. Left ventricular systolic pressure D. Left atrial filling pressure

Explanations (u) A. See D for explanation. (u) B. See D for explanation. (u) C. See D for explanation. (c) D. Pulmonary capillary wedge pressure indirectly measures left atrial filling pressures.

134.

History & Physical/Cardiology A 26 year-old male presents with increased dyspnea with exercise. He has noted a decrease in his exercise tolerance over the past several months. He denies chest pain or skipped heart beats. Echocardiogram reveals left ventricular hypertrophy with asymmetric septal hypertrophy. Ejection fraction is 65%. Which of the following is the most likely presenting history or physical exam finding? A. He has an older brother with the same diagnosis. B. An S3 gallop is heard. C. Patient notes completing a course of adriamycin. D. Elevated jugular venous distension is noted.

Explanations (c) A. Hypertrophic cardiomyopathy can be genetic and present in 25% of first degree relatives. (u) B. Dilated cardiomyopathy may present with an S3 gallop due to volume overload. (u) C. Adriamycin chemotherapy can lead to cardiac dysfunction, dilated cardiomyopathy and eventually heart failure. (u) D. Restrictive or dilated cardiomyopathy may present with JVD due to abnormalities of filling.

135.

Health Maintenance/Cardiology Which of the following lifestyle recommendations would most benefit the heart failure patient's quality of life? A. Begin a regular exercise program B. Total salt restriction C. Home monitoring of blood pressure D. Increase home oxygen requirements

Explanations (c) A. Exercise training improves activity tolerance and deconditioning offering some recompensation. (u) B. Although salt restriction is a recommendation total salt restriction would be unrealistic to achieve. (u) C. Home monitoring of blood pressure will not improve a heart failure patient's quality of life. (u) D. Increasing the requirements for home oxygen could signal that they are worsening and is palliative for their quality of life.

136.

History & Physical/Cardiology A newborn presents with blue discoloration of the peripheral extremities which worsens with exposure to cold and improves with warming. Which of the following is the most likely cause? A. Raynauds phenomenon B. Livedo reticularis C. Erythromelagia D. Acrocyanosis

Explanations (u) A. Raynauds phenomenon has a triphasic color response and is exacerbated by cold or emotions. (u) B. Livedo reticularis is characterized by a lacy pattern on the skin of the lower extremities. (u) C. Erythromelagia is red, painful extremities. (c) D. Acrocyanosis is a blue discoloration of the digits, usually of the peripheral extremities in newborns, which worsens with exposure to cold and improves with warming.

137.

Diagnosis/Cardiology A two month-old infant appeared well until three weeks ago when he became dyspneic and had difficulty feeding. A 4/6 holosystolic murmur is heard at the left lower sternal border in the 3rd ICS. An electrocardiogram (ECG) shows left and right ventricular hypertrophy. Which of the following is the most likely diagnosis? A. Atrial septal defect B. Pulmonary hypertension C. Ventricular septal defect D. Tricuspid insufficiency

Explanations (u) A. An ASD usually presents with a soft mid-systolic murmur in the second left ICS with a widely split and fixed S2. (u) B. With pulmonary hypertension the murmur may be most consistent with pulmonary or tricuspid insufficiency. (c) C. This is a classic presentation for a ventricular septal defect. (u) D. Tricuspid insufficiency is a systolic murmur heard best at the 4th ICS that may radiate to the apex.

138.

Diagnostic Studies/Cardiology Which of the following laboratory tests would be elevated in a patient with symptomatic heart failure? A. Lactate dehydrogenase (LDH) B. Troponin I (TnI) C. C-reactive protein (CRP) D. Brain natriuretic peptide (BNP)

Explanations (u) A. LDH is not a sensitive marker in patients with heart failure. (u) B. Troponin measurements are specific for myocardial infarctions. (u) C. Increased serum levels of CRP are found in patients with unstable angina and MI. They can be a strong predictor of coronary events. (c) D. BNP is expressed in the ventricles and is a sensitive assay in patients with heart failure.

139.

Clinical Therapeutics/Cardiology When prescribing loop diuretics which of the following electrolytes should be most closely monitored? A. Calcium B. Potassium C. Sodium D. Chloride

Explanations (u) A. See B for explanation. (c) B. Potassium along with magnesium should be monitored when prescribing loop diuretics. (u) C. See B for explanation. (u) D. See B for explanation.

140.

Diagnostic Studies/Cardiology Which of the following studies is the best initial diagnostic evaluation to estimate ventricular size and hypertrophy? A. Electrocardiogram (ECG) B. Cardiac CT scan C. Echocardiogram D. Myocardial perfusion imaging

Explanations (u) A. ECG is not sensitive or reliable to estimate ventricular size and hypertrophy. (u) B. Cardiac CT scan can detect coronary calcification, but is most sensitive to assess disorders of the aorta. (c) C. Echocardiogram provides the safest and most reliable means to evaluate ventricular size, hypertrophy and function. (u) D. Myocardial perfusion imaging is used for measurement of LV ejection fraction and assess regional wall motion abnormalities.

141.

Diagnosis/Cardiology A 44 year-old female presents to clinic for evaluation of a syncopal episode that occurred while walking her dog two days ago. She denies amnesia or head trauma. She has had increasing dyspnea on exertion and pedal edema. Physical examination reveals clubbing of her fingers and central cyanosis. Auscultation of the heart reveals tricuspid insufficiency, widely split second heart sound with a palpable P2. Echocardiogram reveals a large ostium secundum atrial septal defect with bidirectional flow. Which of the following is a secondary complication in this patient? A. Left heart failure B. Ebsteins anomaly C. Tricuspid stenosis D. Pulmonary hypertension

Explanations (u) A. This patient would more likely have right heart failure than left heart failure. (u) B. Ebsteins anomaly is apical displacement of the septal tricuspid leaflet and not caused by an ASD. (u) C. Tricuspid stenosis is not caused by an ASD. (c) D. Her symptoms and exam findings are consistent with pulmonary hypertension and in her case, Eisenmengers disease, which is a late finding.

142.

Clinical Intervention/Cardiology Which of the following treatments will most benefit the diabetic patient with two vessel coronary disease? A. Stent placement B. Percutaneous balloon angioplasty C. Medical management D. Coronary artery bypass graft

Explanations (u) A. See D for explanation. (u) B. See D for explanation. (u) C. See D for explanation. (c) D. CABG is the treatment of choice in a diabetic with two or three vessel disease.

143.

Diagnosis/Cardiology A 24 year-old male comes to the clinic with a one week history of pain and swelling that involves the entire right upper extremity. He exercises frequently and has noticed the pain worsening while lifting weights. Examination shows enlarged cutaneous veins over the right anterior chest wall with a palpable cord. His right hand appears dusky. Which of the following is the most likely diagnosis? A. Axillary-subclavian venous thrombosis B. Thromboangiitis obliterans C. Superficial thrombophlebitis of the cephalic vein D. Brachial artery occlusion

Explanations (c) A. Axillary-subclavian venous thrombosis can occur in someone who strenuously exercises, has had a central venous catheter or history of venous thrombosis. (u) B. Thromboangiitis obliterans involves the arteries, although the smaller veins can be included and is linked to tobacco use. (u) C. This presentation is not consistent with superficial thrombophlebitis and there is no history of varicosities or previous IVs. (u) D. Symptoms are not consistent with brachial arterial occlusion.

144.

History & Physical/Cardiology When performing a pre-participation sports physical in the adolescent population, a murmur with which of the following qualities indicates a risk for sudden death during exercise? A. Increases with the Valsalva maneuver B. Increases with squatting maneuver C. Associated with a mid-systolic click D. Mid-systolic without radiation to the carotids

Explanations (c) A. Hypertrophic cardiomyopathy (HCM) is a known cause of sudden death during or just after physical exertion and competitive sports. The murmur associated with HCM is worsened by conditions that cause reduced ventricular volume such as the Valsalva maneuver, sudden standing, and tachycardia. (u) B. Typical systolic flow murmurs will become accentuated with maneuvers which increase venous blood flow to the heart and these murmurs do not place athletes at risk for sudden cardiac death. (u) C. Mitral valve prolapse is the most common type of heart murmur that is associated with a mid-systolic click. Mitral valve prolapse does not place the patient at risk for sudden cardiac death. (u) D. A mid-systolic heart murmur that fails to radiate into the carotids is most commonly associated with a benign systolic flow murmur and does not place the athlete at risk for sudden cardiac death.

145.

Clinical Therapeutics/Cardiology Which of the following antiarrhythmic drugs can be associated with hyper- or hypothyroidism following long-term use? A. Quinidine B. Amiodarone C. Digoxin D. Verapamil

Explanations (u) A. See B for explanation. (c) B. Amiodarone is structurally related to thyroxine and contains iodine, which can induce a hyper- or hypothyroid state. (u) C. See B for explanation. (u) D. See B for explanation.

146.

Diagnosis/Cardiology A 56 year-old male with a known history of polycythemia suddenly complains of pain and paresthesia in the left leg. Physical examination reveals the left leg to be cool to the touch and the toes to be cyanotic. The popliteal pulse is absent by palpation and Doppler. The femoral pulse is absent by palpation but weak with Doppler. The right leg and upper extremities have 2+/4+ pulses throughout. Given these findings what is the most likely diagnosis? A. Venous thrombosis B. Arterialthrombosis C. Thromboangiitis obliterans D. Thrombophlebitis

Explanations (u) A. See B for explanation. (c) B. Arterialthrombosishasoccurredandisevidencedbythelossofthepoplitealanddorsalispedispulse.Thisis a surgical emergency. Venous occlusion and thrombophlebitis do not result in loss of arterial pulse. (u) C. See B for explanation. (u) D. See B for explanation.

147.

History & Physical/Cardiology A 25 year-old female presents with a three day history of chest pain aggravated by coughing and relieved by sitting and leaning forward. She is febrile and a CBC with differential reveals leukocytosis. Which of the following physical examination signs is characteristic of her problem? A. Pulsus paradoxus B. Localized crackles C. Pericardial friction rub D. Wheezing

Explanations (u) A. Pulsus paradoxus is a classic finding for cardiac tamponade. (u) B. Localized crackles are associated with pneumonia and consolidation, not pericarditis. (c) C. Pericardial friction rub is characteristic of an inflammatory pericarditis. (u) D. Wheezing is characteristic for pulmonary disorders, such as asthma.

148.

Diagnosis/Cardiology During physical examination an elderly patient is noted to have a painless, browncolored ulceration in the area of the medial malleolus. Which of the following is the most likely diagnosis? A. Arterial ulcer B. Venous ulcer C. Arterial insufficiency D. Diabetic ulcer

Explanations (u) A. Arterial ulcers typically are the last in the sequence of events of peripheral arterial disease; which include decreased or absent pulses distal to the blockage, muscle atrophy, hair loss, thickened nails, smooth and shiny skin, reduced skin temperature, pallor, cyanosis, ulcers, and gangrene. Arterial ulcers typically occur on the feet in the areas of pressure points. (c) B. Venous ulceration develops in the lower extremity secondary to venous incompetence and chronic edema. The medial aspect of the ankle is the most common location. (u) C. Arterial insufficiency is most likely to present with symptoms of claudication prior to the development of skin ulcers. When these ulcers do occur, they are most commonly seen as arterial ulcers and not venous ulcers. (u) D. Diabetic ulcers typically occur due to atherosclerosis and arterial insufficiency along with diabetic peripheral neuropathy. These ulcers are more likely to occur on pressure points on the foot and fail to heal because of poor circulation to these areas.

149.

Health Maintenance/Cardiology Which of the following is a proven risk factor for the development of abdominal aortic aneurysm? A. Infective endocarditis B. Diabetes mellitus C. Cigarette smoking D. Alcohol abuse

Explanations (u) A. Infective endocarditis is not associated with the development of abdominal aortic aneurysm. (u) B. Diabetic patients do have a higher rate of atherosclerosis, but there is no clear causal evidence of diabetics being at higher risk for the development of abdominal aortic aneurysm. (c) C. Cigarette smoking is the primary risk factor for the development of aortic aneurysms. (u) D. Alcohol abuse is not related to development of abdominal aortic aneurysm.

150.

Clinical Therapeutics/Cardiology When instituting diuretic therapy for patients with heart failure, which of the following is considered the treatment of choice as first-line therapy in a failing kidney due to its improved sodium clearance? A. Hydrochlorothiazide (Diuril) B. Bumetanide (Bumex) C. Spironolactone (Aldactone) D. Acetazolamide (Diamox)

Explanations (u) A. Thiazide diuretics may have better hypertension control than the short acting loop diuretics but they are generally ineffective when the glomerular filtration rate falls below 30-40 mL/min. (c) B. Loop diuretics remain active in severe renal insufficiency and are the most effective type of diuretics used in the management of heart failure symptoms. These agents have a rapid onset and result in natriuresis due to their activity in the ascending limb in the Loop of Henle. (u) C. Potassium-sparing diuretics have very weak diuretic properties and are useful as adjunctive therapy in patients with Stage 3 or Stage 4 heart failure who are already on a first-line diuretic and other agents. (u) D. Acetazolamide is a carbonic anhydrase inhibitor and works by causing a metabolic acidosis with loss of bicarbonate in the failing kidney. It is a weak diuretic and does not work in the setting of a failing kidney.

151.

Diagnostic Studies/Cardiology High ventricular filling pressures are indicated by a rise in which of the following? A. Brain natriuretic peptide B. Troponin C. Myoglobin D. Creatinine

Explanations (c) A. Brain natriuretic peptide (BNP) is a hormone released from the myocardium when stretched such as with high ventricular filling pressures from CHF. It can indicate or be used as prognostic evaluation in patients with acute CHF. (u) B. Troponin is a marker for cardiac muscle damage as in an acute myocardial infarction or ischemia. (u) C. Myoglobin is a byproduct of muscle destruction or damage from ischemia or other causes of skeletal muscle damage. Myoglobin may rise with high ventricular pressures, however it is nonspecific. (u) D. Creatinine is a marker of renal function.

152.

Clinical Intervention/Cardiology Which of the following is first-line treatment for a symptomatic bradyarrhythmia due to sick sinus syndrome? A. Permanent pacemaker B. Radiofrequencyablation C. Antiarrhythmic therapy D. Anticoagulation therapy

Explanations (c) A. Permanent pacemakers are the therapy of choice in patients with symptomatic bradyarrhythmias in sick sinus syndrome. (u) B. Radiofrequency ablation is used for the treatment of accessory pathways in the heart. (u) C. See A for explanation. (u) D. See A for explanation.

153.

Scientific Concepts/Cardiology What is the most likely mechanism responsible for retinal hemorrhages and neurologic complications in a patient with infective endocarditis? A. Metabolicacidosis B. Embolizationofvegetations C. Hypotension and tachycardia D. Activation of the immune system

Explanations (u) A. See B for explanation. (c) B. The vegetations that occur during infective endocarditis can become emboli and can be dispersed throughout the arterial system. (u) C. See B for explanation. (u) D. Glomerulonephritis and arthritis result from activation of the immune system.

154.

Diagnostic Studies/Cardiology Which of the following is a non-invasive quick method of evaluating a patient with suspected lower extremity arterial insufficiency? A. Ankle-Brachial Index B. Striker Tonometry C. CT Angiography D. Lower extremity arteriography

Explanations (c) A. The single most useful index is the ankle pressure. This can be obtained with an Ankle-Brachial Index (ABI) which the severity of signs and symptoms of arterial insufficiency are correlated to the findings on the ABI. It is a non-invasive study that can be performed in an office setting. (u) B. Striker Tonometry is used to evaluate compartment pressures. (u) C. CT angiography is an invasive test involving radiation and contrast that cannot be done in an office setting. (u) D. Lower extremity arteriography is an invasive test involving radiation and contrast that cannot be done in an office setting.

155.

History & Physical/Cardiology Which of the following is the earliest symptom for patients with left ventricular failure? A. Dependentedema B. Dyspneaonexertion C. Congestion D. Chest pain

Explanations (u) A. Right ventricular failure is manifested by dependent edema and congestion in the lungs. (c) B. Patients with left ventricular heart failure may be comfortable at rest and may experience their first symptoms with dyspnea with conversation or with mild exertion. (u) C. Right ventricular failure is manifested by congestion in the lungs with cough being a possible manifestation of this congestion. (u) D. Patients who have on-going left ventricular failure may have underlying coronary heart disease. When the heart failure is progressing, chest pain (manifested by angina) and myocardial infarction may be manifestations of ongoing, progressive heart failure. Acute decline in heart failure may lead to acute myocardial infarction but this tends to be a very late symptom.

156.

Clinical Therapeutics/Cardiology When utilizing medical treatment for hypertension, which of the following classes of medications should be used with caution in those with elevated potassium levels or intrinsic renal disease? A. Beta blockers B. Calcium channel blockers C. Central alpha agonists D. ACE inhibitors

Explanations (u) A. Beta blockers primarily work by decreasing cardiac contraction and slowing the heart rate. There is no association with hyperkalemia. (u) B. Calcium channel blockers (especially the dihydropyridines) act as vasodilators with some effect at lessening cardiac contraction. Their use is not associated with hyperkalemia. (u) C. Central alpha agonists stimulate the alpha receptors in the brain resulting in decreased vessel wall pressures. This process is responsible for causing a lowering of the blood pressure. There is little effect on the kidney and these agents do not cause hyperkalemia. (c) D. ACE inhibitors are recognized as valuable agents in the management of hypertension but they may result in hyperkalemia in patients with intrinsic renal disease. They should be used with caution in patients who are taking potassiumsparing diuretics. These agents reduce glomerular filtration pressure resulting in retention of potassium since less filtration occurs in the kidney.

157.

Clinical Intervention/Cardiology A 60 year-old male has unstable angina, but is otherwise healthy. A 90% lesion is found in the left main coronary artery. Which of the following interventions is most appropriate? A. Thrombolysis with t-PA B. Medical management with nitrates C. Coronary artery bypass graft (CABG) D. Percutaneous transluminal coronary angioplasty

Explanations (u) A. Thrombolysis is recommended in acute embolic occlusion, not chronic. (u) B. Medical management is appropriate only for patients who are not surgical candidates. (c) C. CABG is indicated in patients with stenosis of the left main coronary artery and those with three-vessel coronary artery disease. (u) D. Percutaneous transluminal coronary angioplasty is not the management of choice in left mainstem artery disease because of increased potential complications and mortality.

158.

Scientific Concepts/Cardiology Which of the following pathogens has been linked with the development of acute myocarditis? A. Human papilloma virus B. Rotavirus C. Human Herpes Virus 6 D. Coxsackie B virus

Explanations (u) A. Human papilloma virus most commonly is associated with venereal warts and not myocarditis. (u) B. Rotavirus is primarily responsible for acute diarrhea. (u) C. Human Herpes Virus 6 is the causative organism for Roseola. (c) D. Although associated with a number of infectious and systemic diseases, myocarditis is most frequently the result of a viral infection, with Coxsackie B virus and echovirus being the most frequently implicated in the infection.

159.

Diagnosis/Cardiology A patient is having a routine physical examination. Funduscopic examination reveals AV narrowing and venous nicking. The nasal border of the optic disc appears blurred. Which of the following is the most likely underlying cause? A. Hypertension B. Maculardegeneration C. Retinal detachment D. Diabetes mellitus

Explanations (c) A. Hypertensive retinopathy may cause AV narrowing and venous nicking due to these blood vessels having increased pressures. (u) B. Macular degeneration is associated with the formation of Drusen and neoproliferation. (u) C. Retinal detachment is observed funduscopically by the retina being displaced from its attachment. Patients will present with complaints of floaters in the eye field or abrupt loss of vision if the detachment is complete. (u) D. Diabetes mellitus is most closely associated with neovascularization and microaneurysms as its primary manifestations.

160.

History & Physical/Cardiology A 33 year-old female presents to the office with a complaint of palpitations. There is no history of any significant heart disease in the past and her symptoms begin and end abruptly on their own. Which of the following is the most likely explanation for these symptoms? A. Atrialfibrillation B. Atrialflutter C. Benign supraventricular tachycardia D. Sinus tachycardia

Explanations (u) A. Atrial fibrillation, the most common cause of sustained irregular heart rates, tends to present in an older population or in patients that have underlying heart disease such as mitral stenosis. (u) B. Atrial flutter is not commonly seen in young patients without underlying heart disease although it does tend to be intermittent in nature when it does occur. (c) C. Benign supraventricular tachycardia tends to occur in a young patient without pre-existing heart disease. Symptoms begin and end abruptly without therapy and occurrences happen only intermittently. It is the most likely diagnosis in this setting. (u) D. Sinus tachycardia may occur in a young patient without underlying heart disease but the palpitations and tachycardia tends to start and stop gradually rather than abruptly.

161.

Diagnosis/Cardiology A patient's EKG reveals widened P waves in lead II and large negative deflection of the P wave in lead V1. Which of the following is the most likely underlying cause for this? A. Right atrial enlargement B. Left atrial enlargement C. Right ventricular hypertrophy D. Left ventricular hypertrophy

Explanations (u) A. Right atrial enlargement is found on EKG with peaked P waves in lead II and a large positive deflection on the initial P wave in lead V1. (c) B. Wide P waves in lead II and a deep negative deflection in lead V1 is due to P-mitrale which is caused by left atrial enlargement. (u) C. Right ventricular hypertrophy is noted on the EKG by having a large R wave in lead AvR along with a deep S wave in leads V5 or V6. (u) D. On an EKG, left ventricular hypertrophy is associated with tall R waves in leads V5 and V6, deep S waves in AvR and V1, and tall R waves in AvL and AvF.

162.

Health Maintenance/Cardiology According to the American College of Cardiology/American Heart Association classification of heart failure, which of the following patients fits the Stage B Classification system? A. Asymptomatic patient with no structural disease or patients who are at high risk for the development of heart failure. B. Asymptomatic patient with structural heart disease. C. Symptomatic patient with structural heart disease. D. Patients with refractory symptoms despite intervention.

Explanations (u) A. According to the ACC/AHA 2005 guidelines, patients with risk factors for heart disease but who have yet to develop symptoms are categorized as Stage A. These patients have hypertension and lipid disorders treated along with lifestyle modifications. (c) B. According to the ACC/AHA 2005 guidelines, patients with structural heart disease who have not yet experienced symptoms are classified as Stage B. This is the initial stage in which medication therapy other than just ACE inhibitors are recommended. (u) C. According to the ACC/AHA 2005 guidelines, patients with symptoms and structural heart disease are classified as Stage C. (u) D. According to the ACC/AHA 2005 guidelines, patients with refractory symptoms are classified as Stage D.

163.

Diagnosis/Cardiology A 15 year-old male presents acutely to the office. His legs are cool to the touch. Examination reveals that his pulses and blood pressure are higher in the upper extremities than the lower extremities. Femoral pulses are delayed and weakened. Which of the following is the most likely underlying diagnosis? A. Pheochromocytoma B. Conn's Syndrome C. Cushing'sSyndrome D. Coarctationoftheaorta

Explanations (u) A. Pheochromocytoma is most commonly associated with palpitations and feelings of warmth along with episodic (later sustained) hypertension. (u) B. Conn's Syndrome, also known as primary hyperaldosteronism, is found in a patient with hypertension who has unprovoked hypokalemia. (u) C. Cushing's Syndrome is associated with hypertension but is associated with the typical appearance of sustained elevated cortisol levels such as purple striae, buffalo hump, and central obesity. (c) D. Coarctation of the aorta typically has narrowing of the aorta proximal to the left subclavian artery with resultant high blood pressure in the upper extremities and decreased run off to the lower extremities following this narrowed segment.

164.

Diagnostic Studies/Cardiology Which of the following valvular heart abnormalities will most likely be seen on echocardiography as a complication of acute myocardial infarction? A. Aortic stenosis B. Aortic regurgitation C. Mitral stenosis D. Mitral regurgitation

Explanations (u) A. Aortic stenosis puts additional strain on the left ventricle and contributes to a patient developing an acute myocardial infarction and does not occur as a result of one. (u) B. Aortic regurgitation is not a consequence of acute myocardial infarction and most commonly occurs as a result of an incompetent valve or dilation of the proximal aorta. (u) C. Mitral stenosis most commonly occurs as a complication of rheumatic fever and not because of an acute myocardial infarction. (c) D. In patients with acute myocardial infarction, echocardiogram can show the severity of mitral regurgitation and the presence of ventricular septal defect if one is present. Acute inferior wall myocardial infarction is associated with acute mitral regurgitation due to necrosis of the posterior papillary muscle which is supplied by the right coronary artery.

165.

History & Physical/Cardiology Which of the following conditions is most closely associated with an increased intensity of the P2 heart sound? A. Atrial septal defect B. Aortic stenosis C. Ventricular septal defect D. Mitral valve prolapse

Explanations (c) A. Atrial septal defect can cause a left to right shunt with resultant increased volume in the right ventricle. The net result of this is that the P2 heart sound will be accentuated because of the increased blood flow in the right ventricle and increased force of contraction in the right ventricle to remove this blood. Atrial septal defect will also cause a fixed split S2 heart sound. (u) B. Aortic stenosis primarily affects the left ventricle and causes left ventricular hypertrophy. As the left ventricle contracts harder against increased valvular resistance, a systolic ejection murmur occurs. (u) C. Ventricular septal defect most commonly causes a holosystolic murmur as its most notable ausculatory finding. (u) D. Mitral valve prolapse most commonly is associated with a mid-systolic click with or without a systolic heart murmur.

166.

Clinical Therapeutics/Cardiology Which of the following medication classes is considered first-line therapy for patients with Raynaud's phenomenon? A. Beta blockers B. Calcium channel blockers C. Central alpha agonists D. Oral nitrates

Explanations (u) A. Beta blockers do not cause vasodilation against this vasospasm which makes them less effective as treatment. (c) B. Calcium channel blockers are first line therapy for patients who have uncontrolled symptoms related to Raynaud's phenomenon. They are vasodilating agents which may play a role in preventing the vasospasm that occurs with this disorder. (u) C. See B for explanation. (u) D. Second line agents for Raynaud's phenomenon may include topical but not oral nitrates.

167.

Scientific Concepts/Cardiology Elevated levels of LP(a) (Lipoprotein a) are considered to be a risk factor for coronary artery disease through which of the following proposed mechanisms? A. Direct inhibition of HDL B. Increasing the formation of VLDL cholesterol C. Competes for binding to the plasminogen receptor D. Enhancement of naturally circulating triglycerides

Explanations (u) A. See C for explanation. (u) B. See C for explanation. (c) C. Lp(a) lipoproteins are secreted by the liver, constitute 10% or less of the total plasma lipoprotein mass, possess kringle domains homologous to plasminogen, and are associated with vascular disease risk. Having domains homologous to plasminogen, Lp(a) will compete with actual plasminogen for its receptor sites. Plasminogen activates plasmin, which facilitates degradation of fibrin and matrix components. The main component of LP (a) is LDL, a known risk factor for atherosclerosis. (u) D. See C for explanation.

168.

Clinical Therapeutics/Cardiology Which of the following medication classes is the recommended treatment for patients who have an anterior wall myocardial infarction with poor left ventricular function? A. Beta blockers B. Calcium channel blockers C. Potassium sparing diuretics D. ACE inhibitors

Explanations (u) A. Beta blockers need to be used with caution in a patient with severe left ventricular dysfunction as they will worsen left ventricular contractility and may make this dysfunction worse. They are used, however, in the early stages of chronic heart failure. (u) B. Calcium channel blockers have no proven mortality benefit in patients with myocardial infarctions and left ventricular dysfunction. (u) C. Although potassium sparing diuretics are part of the later stage treatment of congestive heart failure and tend to potentiate the other therapies, they are not first-line therapy in a patient with left ventricular dysfunction. (c) D. ACE inhibitors have been proven to be effective in the therapy of heart failure, especially in the setting of left ventricular dysfunction. They are considered first-line therapy in patients with symptomatic left ventricular systolic function.

169.

Diagnosis/Cardiology The 35 year-old patient presents after a syncopal episode while throwing a football with his son. Examination reveals regular heart rate and EKG is normal. There were no symptoms prior to the episode. Right radial pulse is decreased. Which of the following is the most likely explanation for the syncope? A. Carotid sinus hypersensitivity B. Vasovagal episode C. Cardiac dysrhythmia D. Subclavian steal syndrome

Explanations (u) A. Carotid sinus hypersensitivity typically presents with syncope that is related to turning of the head (such as backing a car out of a driveway) or from tight collars. (u) B. Vasovagal episodes may result in syncope but there would not be abnormal pulse findings. (u) C. Cardiac dysrhythmia may occur in the setting of exercise but this would not produce a decreased pulse on the affected side. (c) D. Subclavian steal syndrome occurs if the subclavian artery is occluded proximal to the origin of the vertebral artery which results in reversal in the direction of blood flow in the ipsilateral vertebral artery. Exercise of the ipsilateral arm may increase demand on the vertebral flow which produces a "subclavian steal".

170.

Scientific Concepts/Cardiology Which of the following is the most common complication that occurs in the setting of acute pericarditis? A. Pericardial effusion B. Left ventricular failure C. Superior vena cava syndrome D. Subclavian steal syndrome

Explanations (c) A. Accumulation of transudate, exudate or blood in the pericardial sac can occur due to pericardial inflammation. (u) B. Patients with acute pericarditis may have problems with filling which affects the right ventricle more than the left ventricle. (u) C. Patients with lung malignancy may develop superior vena cava syndrome as a result of tumor invasion into the superior vena cava. (u) D. Patients with subclavian steal syndrome typically present with arm ischemia and syncope and is not related to pericarditis.

171.

Diagnostic Studies/Cardiology Which diagnostic study is considered to be the strategy of choice for symptomatic patients with recurrent ischemia, hemodynamic instability or impaired left ventricular dysfunction? A. Stress echocardiography B. Exercise treadmill testing C. Coronary angiography D. Cardiac magnetic resonance imaging

Explanations (h) A. Stress echocardiography should not be performed in the setting of a patient who is acutely symptomatic. (h) B. Exercise treadmill testing should not be performed in the setting of an unstable patient with ongoing cardiac symptoms. (c) C. Coronary or cardiac catheterization is the gold standard technique in the evaluation of patients with significant cardiac symptoms. Anatomical information along with degree of coronary artery blockages are provided and patients may be able to undergo coronary revascularization during or after this procedure. (u) D. Cardiac magnetic resonance imaging has limited availability and is not part of national guidelines for evaluation of the cardiac patient.

172.

History & Physical/Cardiology Which of the following is a systemic manifestation of infective endocarditis? A. Hemarthrosis B. Petechiae C. Cafe au lait spots D. Bronzing of the skin

Explanations (u) A. Hemarthrosis is most commonly a consequence of a clotting disorder such as hemophilia. (c) B. Petechiae, splinter hemorrhages, Janeway lesions, and Osler's nodes are systemic manifestations of patients who have infective endocarditis. (u) C. Cafe au lait spots are seen in Neurofibromatosis (von Recklinghausen's syndrome). (u) D. Bronzing of the skin is most commonly associated with hemochromatosis or Addison's disease.

173.

Diagnostic Studies/Cardiology A 76 year-old male presents after returning from a Safari in Africa. Seven days ago he experienced chest pressure lasting one hour that did not respond to three sublingual nitroglycerin tablets. There was no ability to have lab work or an EKG. The pain has not returned. If the patient had a nonSTEMI myocardial infarction, which of the following studies will still be positive? A. Electrocardiogram B. Myoglobulin C. CK-MB index D. Troponin I

Explanations (u) A. Patients suffering from a non-STEMI myocardial infarction will not develop Q waves and most likely will have a normal EKG five days after an acute event. (u) B. Myoglobulin is a nonspecific enzyme that is released into the circulation after any skeletal muscle damage, including a myocardial infarction. It is the first enzyme that becomes positive in the setting of acute myocardial infarction but its non-specific measurement makes it less useful in the setting of acute myocardial infarction. It returns to baseline within 24 hours after infarction. (u) C. CK-MB index has improved sensitivity for myocardial muscle damage that occurs with acute myocardial infarction but it returns to baseline within 2-3 days after injury. (c) D. Troponin I levels will stay positive for at least one week following myocardial infarction and is the preferred enzyme to measure in this setting.

174.

Clinical Intervention/Cardiology A 52 year-old patient with episodes of syncope has an electrocardiogram which shows a consistently prolonged PR interval with a missing QRS every two beats. Which of the following is the most effective management? A. Permanent pacing B. Beta-blocker C. ACEInhibitor D. Defibrillation

Explanations (c) A. This is consistent with ECG findings of a Mobitz type II AV block. Since the patient is symptomatic this type of AV block requires a permanent pacing to prevent total AV disassociation. (u) B. Beta-blockers will slow conduction from the AV node and is not indicated with this type of AV block. (u) C. There is no indication for ACE Inhibitors in Mobitz Type II heart block. (u) D. Defibrillation is not indicated in a person with AV block.

175.

Health Maintenance/Cardiology According to the Joint National Commission VII Guidelines, blood pressure targets are lower in patients with diabetes mellitus and what other condition? A. Liver disease B. Renal disease C. Thyroid disease D. Peripheral vascular disease

Explanations (u) A. See B for explanation. (c) B. Blood pressure targets for hypertensive patients at the greatest risk for cardiovascular events, particularly those with diabetes and chronic kidney disease, are lower (less than 130/80) than for those individuals with lower cardiovascular risk (goal is less than 140/90). (u) C. See B for explanation. (u) D. See B for explanation.

176.

Diagnosis/Cardiology A patient presents to the office following a syncopal episode. The patient claims that the syncope occurs when he changes position such as rolling over in bed or when he bends over to tie his shoes. Which of the following is the most likely explanation for this presentation? A. Carotid sinus hypersensitivity B. Vasovagal episode C. Subclavian steal syndrome D. Atrial myxoma

Explanations (u) A. Carotid sinus hypersensitivity may present with syncope but is usually related to tight collars or when excessively turning the head. (u) B. Vasovagal episodes may occur with syncope as its manifestation but it is not caused by changes in position. (u) C. Subclavian steal syndrome may present with syncope that is related to exercise of the affected arm which results in a decreased pulse when the Adson maneuver is performed. (c) D. Atrial myxoma most commonly presents with sudden onset of symptoms that are typically positional in nature due to the effect that gravity has on the tumor. Myxomas are the most common type of primary cardiac tumor in all age groups and are most commonly found in the atria.

177.

Clinical Therapeutics/Cardiology Which of the following is the optimal therapy for a 76 year-old patient with no allergies who has chronic atrial fibrillation? A. Aspirin B. Clopidogrel (Plavix) C. Warfarin (Coumadin) D. Low molecular weight heparin

Explanations (u) A. Aspirin's role to prevent thromboembolism in atrial fibrillation is limited to patients with no risk factors who are under age 65. (u) B. Clopidogrel is not the optimal therapy for patients with atrial fibrillation. (c) C. Patients older than age 75 who have chronic atrial fibrillation should be anticoagulated with warfarin to maintain an INR between 2.5 and 3.0 for optimum therapy unless a contraindication to therapy exists. (u) D. Due to the increased costs and need for parenteral therapy, daily subcutaneous heparin is not first line therapy unless warfarin therapy is contraindicated.

178.

Clinical Intervention/Cardiology Patients who undergo percutaneous angioplasty or who have coronary artery revascularization often are treated with glycoprotein IIb/IIIa inhibitors. What is the major side effect associated with these agents? A. Hypotension B. Bleeding C. Coronary vasospasm D. Acute renal failure

Explanations (u) A. See B for explanation. (c) B. Glycoprotein IIb/IIa inhibitors have their activity in the final stages of platelet bridging and are associated with bleeding when used in the management of acute myocardial infarction. Since they are effective at treating and preventing new clot formation, bleeding is the main concern and complication with the use of these agents. (u) C. See B for explanation. (u) D. See B for explanation.

179.

Health Maintenance/Cardiology Which of the following is an absolute contraindication for the performance of exercise stress testing for patients who wish to start an exercise program? A. Second degree heart block type 1 B. Severe aortic stenosis C. Atrial fibrillation with controlled ventricular response D. Recent diagnosis of lung cancer

Explanations (u) A. See B for explanation. (c) B. Contraindications to stress testing include rest angina within the last 48 hours, unstable cardiac rhythm, hemodynamically unstable patient, severe aortic stenosis, acute myocarditis, uncontrolled heart failure, and active infective endocarditis. (u) C. See B for explanation. (u) D. See B for explanation.

180.

History & Physical/Cardiology A 23 year-old male with recent upper respiratory symptoms presents complaining of chest pain. His pain is worse lying down and better sitting up and leaning forward. Electrocardiogram shows widespread ST segment elevation. Which of the following is the most likely physical examination finding in this patient? A. Elevated blood pressure B. Subungual hematoma C. Diastolic murmur D. Pericardial friction rub

Explanations (u) A. Acute pericarditis is usually not associated with elevated blood pressure. One would expect to see hypertensive pressures in the setting of an aortic dissection. (u) B. Subungual hematomas are usually seen in endocarditis not pericarditis. (u) C. A diastolic murmur in a patient with chest pain would likely be associated with acute aortic regurgitation in the setting of an aortic dissection. (c) D. This patient has symptoms consistent with acute pericarditis and would most likely have a pericardial friction rub on examination.

181.

Diagnostic Studies/Cardiology A 53 year-old male with history of hypertension presents complaining of recent 4/10 left-sided chest pain with exertion that is relieved with rest. He states the pain usually lasts approximately 4 minutes and is relieved with rest. Heart examination reveals regular rate and rhythm with no S3, S4, or murmur. Lungs are clear to auscultation bilaterally. Electrocardiogram reveals no acute changes. Which of the following is the most appropriate initial step in the evaluation of this patient? A. Cardiac catheterization B. CT Angiogram of the chest C. Echocardiogram D. Nuclear stress test

Explanations (u) A. This patient has signs and symptoms consistent with stable angina. Noninvasive diagnostic testing is preferred in this patient. (u) B. CT angiogram may be useful for the evaluation of chest pain, however its role in routine practice has not been established. (u) C. This patient has signs and symptoms of stable angina. There are no signs of valvular heart disease on examination. While an echocardiogram may be performed at some point, it is not the best initial diagnostic step to determine the etiology of the patient's angina. (c) D. Nuclear stress testing is the most appropriate initial diagnostic study in the evaluation of a patient with signs and symptoms consistent with stable angina.

182.

Clinical Therapeutics/Cardiology A 48 year-old male with diabetes mellitus presents for routine physical examination. Of note his blood pressure each of his last two follow-up visits was 150/90 mmHg. Today the patient's BP is 148/88 mmHg. The patient denies complaints of chest pain, change in vision, or headache. Which of the following is the most appropriate management for this patient? A. Atenolol (Tenormin) B. Nifedipine (Procardia) C. Hydralazine (Apresoline) D. Lisinopril (Zestril)

Explanations (u) A. See D for explanation. (u) B. See D for explanation. (u) C. See D for explanation. (c) D. ACE inhibitors are the first line treatment of choice in a patient with hypertension and diabetes.

183.

Health Maintenance/Cardiology A 36 year-old female presents for a routine physical. She has no current complaints and her only medication is oral contraceptives. The patient is preparing for a trip to Australia and is worried about the long flight as her mom has a history of deep vein thrombosis after a long trip several years ago. Physical examination reveals BP 110/60 mmHg, HR 66 bpm, regular. Heart is regular rate and rhythm without murmur, lungs are clear to auscultation bilaterally and extremities are without edema. Which of the following is the most appropriate recommendation for your patient? A. Discontinue oral contraceptives B. Recommend walking frequently during the flight C. Begin daily aspirin therapy D. Increase fluid intake 2-3 days prior to the flight

Explanations (u) A. See B for explanation. (c) B. The risk of deep vein thrombosis after air travel increases with flight duration. Preventive measures for patients include using support hose and performing in-flight exercises and walking. (u) C. See B for explanation. (u) D. See B for explanation.

184.

Diagnostic Studies/Cardiology A 3 month-old female presents with her mom for physical examination. The patient's mom denies any complaints. On examination you note a welldeveloped, well-nourished infant in no apparent distress. There is no cyanosis noted. Heart examination reveals a normal S1 with a physiologically split S2. There is a grade III/VI high-pitched, harsh, pansystolic murmur heard best at the 3rd and 4th left intercostal spaces with radiation across the precordium. Which of the following is the initial diagnostic study of choice in this patient? A. CT angiogram B. Electrocardiogram C. Echocardiogram D. Cardiac catheterization

Explanations (u) A. This patient has signs and symptoms consistent with a ventricular septal defect (VSD). CT angiogram and electrocardiogram are not indicated in establishing the diagnosis of a VSD. (u) B. See A for explanation. (c) C. Echocardiogram is the initial diagnostic study of choice in the diagnosis of a VSD. (u) D. Cardiac catheterization may be necessary to accurately measure pulmonary pressures or if a VSD can not be well localized on echocardiogram, but it is not the initial diagnostic study of choice.

185.

Clinical Intervention/Cardiology A 20 year-old male presents with complaint of brief episodes of rapid heart beat with a sudden onset and offset that have increased in frequency. He admits to associated shortness of breath and lightheadedness. He denies syncope. Electrocardiogram reveals a delta wave prominent in lead II. Which of the following is the most appropriate long-term management in this patient? A. Implantable cardio defibrillator B. Radiofrequency ablation C. Verapamil (Calan) D. Metoprolol (Lopressor)

Explanations (u) A. Implantable cardio defibrillators are indicated in the treatment of ventricular arrhythmias, not Wolf-Parkinson- White (WPW) syndrome. (c) B. Radiofrequency ablation is the procedure of choice for long-term management in patients with accessory pathways (WPW) and recurrent symptoms. (u) C. Calcium channel blockers and beta-blockers are not the best options for the long-term management of WPW. They may decrease the refractoriness of the accessory pathway or increase the refractoriness of the AV node in patients with atrial fibrillation or atrial flutter who have an antegrade conducting bypass tract. This may lead to faster ventricular rates. (u) D. See C for explanation.

186.

Diagnosis/Cardiology A 60 year-old female recently discharged after an 8 day hospital stay for pneumonia presents complaining of pain and redness in her right arm. The patient thinks this was the area where her IV was placed. The patient denies fever or chills. Examination of the area reveals localized induration, erythema and tenderness. There is no edema or streaking noted. Which of the following is the most likely diagnosis? A. Acute thromboembolism B. Thrombophlebitis C. Cellulitis D. Lymphangitis

Explanations (u) A. Acute thromboembolism is usually associated with edema of the extremity and warm temperature. Thromboembolism is unusual after thrombophlebitis. (c) B. This patient's signs and symptoms are consistent with thrombophlebitis. Short-term venous catheterization of a superficial arm vein is commonly the cause and thrombophlebitis characterized by dull pain, induration, redness and tenderness along the course of the vein. (u) C. Cellulitis is usually associated with fever, increased warmth over the affected area and associated edema. (u) D. Lymphangitis is associated with fever, malaise, chills, and streaking.

187.

History & Physical/Cardiology A patient with a history of chronic venous insufficiency presents for routine follow-up. Which of the following findings is most likely on physical examination? A. Cold lower extremities B. Diminished pulses C. Lower extremity edema D. Palpable cord

Explanations (u) A. Cold lower extremities are more commonly seen in peripheral arterial, not venous, disease. (u) B. Diminished pulses are seen in peripheral arterial disease. (c) C. Patients with chronic venous insufficiency will commonly have lower extremity edema. (u) D. A palpable cord is more common in superficial thrombophlebitis.

188.

Diagnostic Studies/Cardiology A 29 year-old female with history of IV drug abuse presents with ongoing fevers for three weeks. She complains of fatigue, worsening dyspnea on exertion and arthralgias. Physical examination reveals a BP of 130/60 mmHg, HR 90 bpm, regular, RR 18, unlabored. Petechiae are noted beneath her fingernails. Fundoscopic examination reveals exudative lesions in the retina. Heart examination shows regular rate and rhythm, there is a grade II-III/VI systolic murmur noted, with no S3 or S4. Lungs are clear to auscultation bilaterally, and the extremities are without edema. Which of the following is the diagnostic study of choice in this patient? A. Electrocardiogram B. CT angiogram of the chest C. Cardiac catheterization D. Transesophageal echocardiogram

Explanations (u) A. See D for explanation. (u) B. See D for explanation. (u) C. See D for explanation. (c) D. This patient's signs and symptoms are consistent with infective endocarditis. The diagnostic study of choice would be a transesophageal echocardiogram.

189.

Clinical Therapeutics/Cardiology A 49 year-old female presents complaining of several episodes of chest pain recently. Initial ECG in the emergency department shows no acute changes. Two hours later, while the patient was having pain, repeat electrocardiogram revealed ST segment elevation in leads II, III, and AVF. Cardiac catheterization shows no significant obstruction of the coronary arteries. Which of the following is the treatment of choice in this patient? A. Nifedipine (Procardia) B. Metoprolol (Lopressor) C. Lisinopril (Zestril) D. Carvedilol (Coreg)

Explanations (c) A. This patient is most likely having coronary artery spasm. This can be treated prophylactically with calcium channel blockers such as nifedipine. (h) B. Beta-blockers may exacerbate the symptoms of coronary vasospasm. (u) C. ACE inhibitors are not effective in the treatment or prevention of coronary vasospasm. (u) D. Carvedilol is not effective in the treatment or prevention of coronary vasospasm.

190.

History & Physical/Cardiology A 75 year-old female with a history of long-standing hypertension presents with shortness of breath. On examination you note a diastolic murmur at the left upper sternal border. Which of the following maneuvers would accentuate this murmur? A. Sitting up and leaning forward B. Lying on left side C. Performing Valsalva maneuver D. Standing upright

Explanations (c) A. This patient has history findings consistent with aortic insufficiency which is characterized by a diastolic murmur that is accentuated when the patient sits up and leans forward. (u) B. The left lateral decubitus position accentuates the murmur of mitral stenosis. (u) C. Valsalva and standing maneuvers help to differentiate the murmurs associated with aortic stenosis and hypertrophic cardiomyopathy. (u) D. See C for explanation.

191.

Diagnosis/Cardiology A 50 year-old male with history of alcohol abuse presents with complaint of worsening dyspnea. Physical examination reveals bibasilar rales, elevated jugular venous pressure, an S3 and lower extremity edema. Chest x-ray reveals pulmonary congestion and cardiomegaly. Electrocardiogram shows frequent ventricular ectopy. Echocardiogram shows left ventricular dilatation and an ejection fraction of 30%. Which of the following is the most likely diagnosis in this patient? A. Hypertrophic cardiomyopathy B. Dilated cardiomyopathy C. Restrictive cardiomyopathy D. Tako-Tsubo cardiomyopathy

Explanations (u) A. Hypertrophic cardiomyopathy is characterized by a hyperdynamic left ventricle with asymmetric left ventricular hypertrophy. (c) B. Dilated cardiomyopathy is often caused by chronic alcohol use. It is characterized by signs and symptoms of left-sided heart failure, a dilated left ventricle and decreased ejection fraction. (u) C. Restrictive cardiomyopathy is characterized more commonly by right-sided heart failure than by left-sided heart failure. There is rapid early filling with diastolic dysfunction. Patients with restrictive cardiomyopathy will have a small thickened left ventricle and a normal or near normal ejection fraction on echocardiogram. (u) D. Tako-Tsubo cardiomyopathy (broken heart syndrome) is characterized by signs and symptoms of acute coronary syndrome, ST segment elevation on ECG and left ventricular apical dyskinesia.

192.

Clinical Therapeutics/Cardiology A 76 year-old active female with history of hypertension and hypothyroidism presents with complaints of palpitations and dyspnea on exertion. On examination vital signs are BP 120/80 mmHg, HR 76 bpm, irregular, RR 16. Heart examination reveals an irregularly, irregular rhythm without murmur. Lungs are clear to auscultation and extremities are without edema. Which of the following is the most important medication to initiate for chronic therapy in this patient? A. Warfarin (Coumadin) B. Verapamil (Calan) C. Amiodarone (Cordarone) D. Digoxin (Lanoxin)

Explanations (c) A. Anticoagulation is necessary in all patients with atrial fibrillation to prevent thromboembolic events unless there is contraindication. (u) B. This patient currently has a controlled ventricular rates and does not require chronic calcium channel blockers or digoxin at this time. (u) C. Antiarrhythmic therapy may be indicated in some patients with atrial fibrillation, but anticoagulation is indicated in all patients unless there is contraindication. (u) D. See B for explanation.

193.

Diagnosis/Cardiology A 58-year old male presents for a six week follow-up after an acute anterior wall myocardial infarction. He denies chest pain and shortness of breath. Electrocardiogram shows persistent ST segment elevation in the anterior leads. Echocardiogram reveals a sharply delineated area of scar that bulges paradoxically during systole. Which of the following is the most likely diagnosis in this patient? A. Left ventricular aneurysm B. Postinfarction ischemia C. Ischemic cardiomyopathy D. Constrictive pericarditis

Explanations (c) A. Left ventricular (LV) aneurysm develops in about 1020 percent of patients following acute myocardial infarctions, especially anterior wall myocardial infarctions. LV aneurysm is identified by ST segment elevation that is present beyond 4-8 weeks after the acute infarct and a scar that bulges paradoxically during systole on echocardiogram. (u) B. Postinfarction ischemia is recurrent ischemia that is more common in patients with non-ST segment elevation myocardial infarctions and is characterized by postinfarction angina. This patient denies any chest pain. (u) C. Ischemic cardiomyopathy would be characterized by decreased ejection fraction on echocardiogram and wall motion abnormalities. Ischemic cardiomyopathy is not associated with ST segment elevation or bulge of scar on echocardiogram. (u) D. Constrictive pericarditis is characterized by signs and symptoms of right-sided heart failure with increased jugular venous pressures and a septal bounce on echocardiogram.

194.

History & Physical/Cardiology A 75 year-old female with history of coronary artery disease and dyslipidemia presents for routine follow-up. Physical examination reveals loss of hair on the lower extremities bilaterally with thinning of the skin. Femoral pulses are +2/4 bilaterally, pedal pulses are diminished bilaterally. Ankle brachial index is reduced. Which of the following signs or symptoms is this patient most likely to have? A. Lower extremity edema B. Calf pain with walking C. Numbness of the lower extremities D. Itching of the lower extremities

Explanations (u) A. This patient has signs and symptoms consistent with arterial insufficiency. Lower extremity edema is seen in patients with venous insufficiency. (c) B. This patient has signs and symptoms consistent with arterial insufficiency and would most likely complain of intermittent claudication. (u) C. Numbness of the lower extremities would be seen with acute arterial occlusion. (u) D. Itching of the lower extremities may be seen in chronic venous insufficiency because of secondary skin changes, but is not common in arterial insufficiency.

195.

Diagnosis/Cardiology A 55 year-old male with history of hypertension and diabetes mellitus presents to the emergency department. The patient's wife states that the patient developed progressive irritability and confusion today after complaining of a headache. Physical examination reveals a BP of 230/130 mmHg and papilledema. Which of the following is the most accurate diagnosis in this patient? A. Resistant hypertension B. Hypertensive urgency C. Hypertensive emergency D. Malignant hypertension

Explanations (u) A. Resistant hypertension is the failure to reach blood pressure control in patients who are compliant with a 3 drug regimen including a diuretic. (u) B. Hypertensive urgency is a systolic BP > 220 or a diastolic BP > 125 in a patient who is asymptomatic or who has disk edema, progressive target organ complications. Hypertensive urgency must be treated within a few hours of presentation. (u) C. Hypertensive emergency is similar to hypertensive urgency, however the BP is significantly elevated and must be lowered within an hour. (c) D. Malignant hypertension is significantly elevated BP with progressive retinopathy, including papilledema, encephalopathy, and headache.

196.

Scientific Concepts/Cardiology A patient undergoes biopsy for suspected myocarditis. Which of the following is the most likely etiologic agent? A. West Nile virus B. Rhinovirus C. Coxsackie B virus D. Cytomegalovirus

Explanations (u) A. See C for explanation. (u) B. See C for explanation. (c) C. Coxsackie B virus, Hepatitis C, adenovirus, and HIV are the predominant agents in clinically significant acute viral myocarditis in the US. (u) D. See C for explanation.

197.

Health Maintenance/Cardiology A 65 year-old male with coronary artery disease, hypertension, and diabetes mellitus is admitted with dyspnea and lower extremity edema. The chest x-ray reveals small bilateral pleural effusions. Echocardiogram shows an ejection fraction of 30% with no valvular heart disease. The patient is treated in the hospital with furosemide (Lasix) and lisinopril (Zestril). What education should be given to this patient upon discharge to help prevent readmission? A. Elevate the head of bed at home B. Avoid physical activity C. Monitor daily weights D. Restrict fluid intake

Explanations (u) A. Elevation of the head of the bed may help the patient if they have symptoms of dyspnea but it will not help prevent readmission to the hospital with a CHF exacerbation. (u) B. In stable patients increasing physical activity or regular exercise can diminish symptoms. (c) C. Strategies to prevent rehospitalization can include monitoring daily weights, case management and patient education regarding selfadjustment of diuretics. (u) D. Fluid restriction is not helpful in the readmission for CHF.

198.

History & Physical/Cardiology A 60 year-old male nonsmoker with history of coronary artery disease presents with complaint of worsening dyspnea on exertion for three weeks. He admits to orthopnea and lower extremity edema, but denies chest pain, palpitations, and syncope. The patient's last echocardiogram revealed an ejection fraction of 30%. Which of the following would you most likely find on physical examination? A. Pericardial friction rub B. Third heart sound C. Accentuated first heart sound D. Mid-systolic click

Explanations (u) A. A pericardial friction rub is a sign of pericarditis, not heart failure. (c) B. An S3 on physical examination is consistent with heart failure. (u) C. An accentuated first heart sound is noted in tachycardia, short PR interval rhythms, increased cardiac output states and mitral stenosis. (u) D. A mid-systolic click is noted in patients with mitral valve prolapse.

199.

Clinical Therapeutics/Cardiology A 78 year-old male with history of coronary artery disease s/p coronary artery bypass grafting, hypertension, and dyslipidemia presents for routine physical examination. He feels well except for occasional brief episodes of substernal chest pain with exertion that are relieved with rest. He denies associated dyspnea, nausea or diaphoresis. Physical examination reveals a BP of 110/70 mmHg, HR 56 bpm, regular, RR 14, unlabored. Lungs are clear to auscultation, heart is bradycardic, but regular with no S3, S4 or murmur. Electrocardiogram done in the office shows no acute ST-T wave changes. Which therapy is indicated for the acute management of this patient's symptoms? A. Sublingual nitroglycerine B. Metoprolol (Lopressor) C. Verapamil (Calan) D. Lisinopril (Zestril)

Explanations (c) A. Sublingual nitroglycerine is the drug of choice for the acute management of chronic stable angina. (u) B. Beta-blockers are preventative and not the first choice for the acute management of chronic stable angina. Betablockers may worsen this patient's bradycardia. (u) C. Calcium channel blockers are the third-line antiischemic agent and may also reduce the patient's heart rate. (u) D. ACE inhibitors will not provide acute relief of anginal symptoms.

200.

Scientific Concepts/Cardiology A 36 year-old male presents for follow-up of his hypertension. The patient is currently on three anti-hypertensive medications without improvement of his blood pressure. On examination his BP is 170/86mmHg and his HR is 60bpm and regular. His heart examination reveals a regular rate and rhythm without S3, S4 or murmur and his lungs are clear to auscultation bilaterally. Abdominal examination reveals a bruit over his left upper abdomen. Which of the following is the most likely underlying etiology for this patient's hypertension? A. Pheochromocytoma B. Renal artery stenosis C. Cushing syndrome D. Coarctation of the aorta

Explanations (u) A. Pheochromocytoma is an uncommon cause of hypertension characterized by paroxysms of headache, sweating and palpitations. There are no bruits associated with pheochromocytoma. (c) B. Renal artery stenosis is characterized by hypertension that is resistant to three or more medications and renal artery bruits on examination. (u) C. Cushing syndrome is characterized by "moon" facies, a buffalo hump, a protuberant abdomen, weakness and headache. There are no renal artery bruits associated with Cushing syndrome. (u) D. Coarctation of the aorta is associated with hypertension in the upper extremities and normal or low blood pressure in the lower extremities. There are often weak femoral pulses and a late systolic ejection murmur or associated aortic insufficiency murmur.

201.

Clinical Therapeutics/Cardiology A 16 year-old male presents with complaint of syncope after basketball practice today. Physical examination reveals a systolic murmur along the left sternal border that increases with Valsalva maneuver. An electrocardiogram reveals left ventricular hypertrophy. Echocardiogram shows asymmetric left ventricular hypertrophy with a hypercontractile left ventricle. Which of the following is the initial medication of choice in this patient? A. Metoprolol (Lopressor) B. Losartan (Cozaar) C. Lisinopril (Zestril) D. Hydrochlorothiazide (Diuril)

Explanations (c) A. Beta-blockers are the initial drug of choice in a symptomatic patient with hypertrophic cardiomyopathy. (u) B. See A for explanation. (u) C. See A for explanation. (u) D. See A for explanation.

202.

Clinical Intervention/Cardiology An 80 year-old female presents with syncope and recent fatigue and lightheadedness over the past month. She denies chest pain or dyspnea. Physical examination reveals BP 130/70 mmHg, HR 40 bpm, regular, and RR 16. Electrocardiogram reveals two p waves before each QRS complex. Which of the following is the treatment of choice for this patient? A. Cardio defibrillator insertion B. Atropine as needed C. Permanent dual chamber pacemaker insertion D. Ritalin therapy daily

Explanations (u) A. Cardio defibrillators treat ventricular tachycardia and are not indicated in the management of second degree AV block. (u) B. Atropine can be used in the acute management of second degree AV block Mobitz type II, but it should not be used as long-term therapy. (c) C. This patient has findings consistent with symptomatic second degree AV block Mobitz type II for which permanent pacing is the treatment of choice. (u) D. Ritalin therapy is not indicated in the management of second degree heart block.

203.

History & Physical/Cardiology Which of the following would be expected on physical examination of a newborn diagnosed with Tetrology of Fallot? A. Palpable right ventricular lift B. Pulsediscrepancybetweenarmsandlegs C. Mid-diastolic murmur with opening snap D. Polymorphous exanthema

Explanations (c) A. Tetralogy of Fallot is commonly associated with a palpable right ventricular lift. (u) B. Coarctation of the aorta is associated with a pulse discrepancy between the upper and lower extremities. (u) C. A mid-diastolic murmur with an opening snap is heard in a patient with mitral stenosis, not Tetralogy of Fallot. (u) D. Polymorphous exanthema is seen in patients with Kawasaki disease.

204.

Clinical Therapeutics/Cardiology A 67 year-old male with history of mitral valve stenosis undergoes a mechanical valve replacement. Which of the following is the appropriate duration of anticoagulation therapy if the patient has no other risk factors for thromboembolic events or significant bleeding risks? A. One month B. Three months C. Six months D. Lifelong

Explanations (u) A. See D for explanation. (u) B. See D for explanation. (u) C. See D for explanation. (c) D. Patients with mechanical valves require lifelong anticoagulation to prevent thrombosis.

205.

Diagnosis/Cardiology A 60 year-old female with history of radiation therapy for the treatment of cancer presents with progressive dyspnea and fatigue. On examination the patient has lower extremity edema, significant ascites, and an elevated jugular venous pressure that does not fall with inspiration. Heart examination reveals a pericardial knock. Echocardiogram shows rapid early filling and reduced mitral inflow velocities with inspiration. Which of the following is the most likely diagnosis in this patient? A. Pulmonaryhypertension B. Atrialmyxoma C. Constrictive pericarditis D. Tako-Tsubo cardiomyopathy

Explanations (u) A. Pulmonary hypertension is usually associated with chest pain, dyspnea, fatigue and syncope. Examination would reveal a narrow splitting of S2 with a loud pulmonic component. Echocardiogram would show increased pulmonary artery pressures, right ventricular enlargement and possibly paradoxical motion of the intraventricular septum. (u) B. Atrial myxoma is characterized by fever, weight loss, malaise, embolization, and a diastolic sound or murmur (tumor plop) on examination. Atrial myxoma would be seen on echocardiogram. (c) C. Constrictive pericarditis is associated with TB, radiation therapy, cardiac surgery, or following viral pericarditis. There is evidence of right-sided heart failure, a positive Kussmaul sign, and a septal bounce and reduced mitral inflow velocities with inspiration on echocardiogram. (u) D. Tako-Tsubo cardiomyopathy (broken heart syndrome) is commonly seen in postmenopausal women who experience signs and symptoms of acute coronary syndrome. Echocardiogram shows left ventricular apical dyskinesia.

206.

Scientific Concepts/Cardiology A 59 year-old male with history of hypertension and dyslipidemia presents with complaint of substernal chest pain for two hours. The pain woke him from sleep, does not radiate, and is associated with nausea and diaphoresis. Electrocardiogram reveals ST segment elevation in leads II, III, and AVF. Which of the following walls of the ventricle is most likely at risk? A. Anterior B. Inferior C. Lateral D. Posterior

Explanations (u) A. See B for explanation. (c) B. Inferior wall myocardial infarction is characterized by ST segment elevation in leads II, III and AVF. (u) C. See B for explanation. (u) D. See B for explanation.

207.

Diagnosis/Cardiology A 24 year-old female presents complaining of palpitations described as occasional "skipped" beats. The patient denies chest pain, lightheadedness, syncope, or dyspnea. On examination you note a midsystolic click without murmur. Which of the following is the most likely diagnosis in this patient? A. Mitral valve prolapse B. Aortic stenosis C. Atrial septal defect D. Pulmonic stenosis

Explanations (c) A. Patients with mitral valve prolapse will often present with complaint of palpitations. Auscultation would reveal a mid-systolic click with or without a late systolic murmur. (u) B. Aortic stenosis presents with a systolic murmur with no click. Patients may complain of chest pain, dyspnea or syncope. (u) C. Atrial septal defect is not associated with a midsystolic click. (u) D. Pulmonic stenosis is not associated with a midsystolic click.

208.

Health Maintenance/Cardiology A 10 year-old female experiences fever and polyarthralgia. On examination you note a new early diastolic murmur. Laboratory results are positive for antistreptolysin O. The patient has no known drug allergies. Which of the following is the recommended prophylaxis for this condition? A. Doxycycline B. Erythromycin C. BenzathinepenicillinG D. Trimethoprim/sulfamethoxazole

Explanations (u) A. Doxycycline and Bactrim are not indicated for the prophylaxis of recurrent rheumatic fever. (u) B. Erythromycin is considered second line for prophylaxis of recurrent rheumatic fever in a patient with a penicillin allergy. (c) C. Recurrences of rheumatic fever are most common in patients who have had carditis during their initial episode and in children. The preferred method of prophylaxis is Benzathine penicillin G every four weeks. (u) D. See A for explanation.

209.

Clinical Intervention/Cardiology A 26 year-old patient is brought to the emergency department after a head on collision. The patient complains of chest pain, dyspnea and cough. Examination reveals the patient to be tachypneic and tachycardic with a narrow pulse pressure. Jugular venous distension is noted. Electrocardiogram reveals nonspecific t wave changes and electrical alternans. Which of the following is the most appropriate management plan for this patient? A. Serial echocardiograms B. Pericardiocentesis C. Cardiac catheterization D. Pericardiectomy

Explanations (u) A. Serial echocardiograms would be indicated if a patient had a small pericardial effusion and no intervention was immediately needed. This patient has signs and symptoms of cardiac tamponade and needs immediate intervention. (c) B. Urgent pericardiocentesis is the initial treatment of choice in a patient with cardiac tamponade. (u) C. There is no indication for cardiac catheterization in the management of cardiac tamponade. (u) D. A partial pericardiectomy may be needed in patients with recurrent pericardial effusions that occur secondary to neoplastic disease and uremia, but there is no indication for partial pericardiectomy in the acute management of cardiac tamponade.

210.

Health Maintenance/Cardiology A 40 year-old G3P3003 female presents complaining of dull aching discomfort of her lower extremities, which is worse in the evening. The patient currently works as a waitress. Examination reveals dilated, tortuous veins beneath the skin in the thigh and leg bilaterally. Which of the following is the best initial approach to prevent progression of disease and complications in this patient? A. Compression stockings B. Warfarin (Coumadin) therapy C. Sclerotherapy D. Clopidogrel (Plavix)

Explanations (c) A. Graduated compression stockings can be used in patients with early varicosities to prevent progression of the disease and when used with leg elevation complications from varicose veins can be avoided. (u) B. There is no indication for warfarin or clopidogrel therapy in patins with varicose veins. (u) C. Scierotherapy is not the best initial choice to prevent disease progression and complications (u) D. See B for explanation.

211.

Diagnostic Studies/Cardiology A 60 year-old male with history of hypertension presents for routine physical examination. He has no current complaints. Vital signs are BP of 136/70 mmHg, HR 60 bpm, regular, RR 14, unlabored. Heart shows regular rate and rhythm with no S3, S4 or murmur, Lungs are clear to auscultation bilaterally, and the abdomen is soft, nontender. There is a 5cm palpable pulsatile abdominal mass noted. Which of the following is the best initial diagnostic study in this patient? A. Magnetic resonance imaging (MRI) B. Arteriography C. Ultrasound (US) D. Plain film

Explanations (u) A. MRI, arteriography, and abdominal flat plate are not indicated in the initial diagnostic evaluation of a patient with a suspected abdominal aortic aneurysm. (u) B. See A for explanation. (c) C. Abdominal ultrasound is the diagnostic study of choice for the initial diagnosis of an abdominal aortic aneurysm. (u) D. See A for explanation.

212.

Diagnosis/Cardiology A 70 year-old female with history of hypertension, diabetes, and hypothyroidism presents with complaint of sudden onset of left lower extremity pain. Examination reveals a cool left lower extremity with a mottled appearance. Dorsalis pedis and posterior tibialis pulses are absent. Which of the following is the most likely diagnosis? A. Acute arterial occlusion B. Thromboangiitis obliterans C. Deep vein thrombosis D. Peripheral neuropathy

Explanations (c) A. Acute arterial occlusion presents with sudden onset of extremity pain, with absent or diminished pulses. The extremity will be cool to the touch and have a mottled appearance. (u) B. Thromboangitis obliterans occurs in younger patients and primarily effects the distal extremities, especially the toes. It is typically secondary to smoking. (u) C. Deep vein thrombosis presents with lower extremity pain and edema. Pulses would be intact. (u) D. Patients with peripheral neuropathy would have diminished sensation. They would not have a mottled appearance and pulses would be intact.

213.

A 45 year-old male presents to the Emergency Department complaining of sudden onset of tearing chest pain radiating to his back. On examination the patient is hypertensive and his peripheral pulses are diminished. Electrocardiogram shows no acute ST-T wave changes. Which of the following is the diagnostic study of choice in this patient? A. Computed tomography (CT) scan B. Transthoracic echocardiogram C. Magnetic resonance imaging (MRI) D. Cardiac catheterization

Explanations (c) A. This patient has signs and symptoms of acute aortic dissection for which CT scan is the diagnostic study of choice. (u) B CT scan is better than transthoracic echocardiogram for the diagnosis of acute aortic dissection. Transesophageal echocardiogram (TEE) is a good diagnostic modality, however it is not always available in the acute setting. (u) C.MRI is good in the diagnosis of a chronic aortic dissection, but the longer imaging time and the difficulty in monitoring the patient during the test makes it not the first choice in the setting of an acute dissection. (u) D. Cardiac catheterization is not indicated in the diagnosis of an acute aortic dissection.

214.

Scientific Concepts/Cardiology A 60 year-old male complains of progressive fatigue and dyspnea. On examination his lungs are clear to auscultation bilaterally, heart exam reveals regular rate and rhythm without S3, S4 or murmur, and extremities show 1+ edema bilaterally. Chest x-ray reveals cardiomegaly. electrocardiogram shows low voltage, and echocardiogram shows an ejection fraction of 55% with a small, thickened left ventricle that has rapid early filling with diastolic dysfunction. Which of the following is the most likely underlying etiology of this patient's cardiomyopathy? A. Alcoholism B. Myocarditis C. Amyloidosis D. Chronic hypertension

Explanations (u) A. Chronic alcohol use is commonly associated with a dilated left ventricle with left ventricular dysfunction. (u) B. Myocarditis is associated with a dilated, not small, left ventricle. (c) C. Amyloidosis is the most common cause of restrictive cardiomyopathy and is associated with a small thickened left ventricle that has rapid early filling with diastolic dysfunction. (u) D. Chronic hypertension is associated with a hypertrophic, hypercontractile left ventricle.

215.

Clinical Intervention/Cardiology A 70 year-old male with history of ischemic cardiomyopathy presents with a syncopal episode. He denies complaints of chest pain, palpitations, or dyspnea. ECG shows no acute ST-T wave changes. Echocardiogram reveals an ejection fraction of 25% with no valvular abnormalities. Which of the following is the most appropriate management for this patient? A. Dual chamber permanent pacemaker B. Diltiazem (Cardizem) C. Implantable cardio defibrillator D. Midodrine (ProAmatine)

Explanations (u) A. See C for explanation. (u) B. See C for explanation. (c) C. This patient has ischemic cardiomyopathy and syncope, which is most likely due to ventricular tachycardia. Instertion of a cardio defibrillator is the management of choice in this patient. (u) D. See C for explanation.

View more...

Comments

Copyright ©2017 KUPDF Inc.
SUPPORT KUPDF